Sei sulla pagina 1di 62

CONTEDO

AOS LEITORES XV OLIMPADA DE MATEMTICA DO CONE SUL Enunciados e Solues XLV OLIMPADA INTERNACIONAL DE MATEMTICA Enunciados e Solues

2 3 13

ARTIGOS
CENTRO DE MASSA E APLICAES GEOMETRIA Emanuel Carneiro & Frederico Giro SEQNCIA DE FIBONACCI Ccero Thiago B. Magalhes COMO QUE FAZ? SOLUES DE PROBLEMAS PROPOSTOS PROBLEMAS PROPOSTOS COORDENADORES REGIONAIS 29

38 43 46 60 62

Sociedade Brasileira de Matemtica

AOS LEITORES
Chegamos a este nmero 21 com dois artigos: um de geometria e outro sobre a seqncia de Fibonacci. So as primeiras publicaes dos respectivos autores na Eureka!. Esperamos que venham outras e que cada vez mais autores contribuam com a revista. Apresentamos as solues da Olimpada do Cone Sul e da Olimpada Internacional de 2004, competies nas quais as equipes brasileiras tiveram muito bom desempenho. Agradecemos mais uma vez as contribuies dos leitores para a seo "Como que faz?" e para a seo dos problemas propostos, com solues e novos problemas, que fazem da Eureka! cada vez mais uma obra de criao coletiva. Aproveitamos para registrar que foi criada em 2004 a Associao Olimpada Brasileira de Matemtica AOBM, uma pessoa jurdica destinada a ajudar as Olimpadas de Matemtica no Brasil a crescerem e se consolidarem. Esperamos que a AOBM sirva como instrumento para maior integrao e organizao da comunidade olmpica. Estimulamos nossos leitores a se associarem AOBM, cujos scios recebem gratuitamente a revista Eureka!. As informaes sobre como se associar AOBM podem ser encontradas na nossa pgina na internet: www.obm.org.br

Os editores

EUREKA! N21, 2005

Sociedade Brasileira de Matemtica

XV OLIMPADA DE MATEMTICA DO CONE SUL


Enunciados e Solues
PROBLEMA 1

Maxi escolheu 3 dgitos e, fazendo todas as permutaes possveis, obteve 6 nmeros distintos, cada um com 3 dgitos. Se exatamente um dos nmeros que Maxi obteve um quadrado perfeito e exatamente trs so primos, encontrar os 3 dgitos que Maxi escolheu. D todas as possibilidades para os 3 dgitos.
SOLUO DE LEANDRO FARIAS MAIA (FORTALEZA CE)

Sejam x1 , x2 , x3 os dgitos escolhidos por Maxi. Note que x1 x2 , x1 x3 , x2 x3 pois as 6 reordenaes: x1 x2 x3 ,..., x3 x2 x1 so distintas. Agora faremos a lista de todos os nmeros de 3 dgitos quadrados perfeitos: 10 2 = 100 202 = 400 302 = 900 2 2 11 = 121 21 = 441 312 = 961 2 2 12 = 144 22 = 484 132 = 169 14 2 = 196 152 = 225 16 2 = 256 17 2 = 289 182 = 324 232 = 529 242 = 576 252 = 625 262 = 676 27 2 = 729 282 = 784

19 2 = 361 292 = 841 Perceba que: Os nmeros que tm algum zero no satisfazem o enunciado: 10 2 , 20 2 ,30 2. Reordenando 1, 6, 9 podemos obter 132 ,14 2 ,312. Assim, 132 ,14 2 ,312 no satisfazem o enunciado. O nmero dever apresentar no mnimo 2 dgitos mpares, pois se tiver no mximo um, teremos (se tivermos) no mximo 2 nmeros primos. Assim; 16 2 ,17 2 ,182 , 252 , 28 2 , 29 2 no satisfazem o enunciado. Os dgitos so distintos. Assim, 112 ,12 2 ,152 , 212 , 22 2 , 26 2 no satisfazem o enunciado.
EUREKA! N21, 2005

Sociedade Brasileira de Matemtica

Nos restam os nmeros: 19 2 , 232 , 24 2 , 27 2. Reordenando: 19 2 = 361;136,163,316,361,613, 631. 232 = 529; 259, 295,529,592,925,952. 24 2 = 576;567,576, 657,675,756,765. 27 2 = 729; 279, 297,729, 792,927,972. Perceba que os dgitos: 1, 3, 6 satisfazem o enunciado, pois, 163, 613, 631 (apenas) so primos e 361 (apenas) quadrado perfeito. 2, 5, 9 no satisfazem o enunciado, pois, 295, 592, 925, 952 so compostos e 529 (apenas) quadrado perfeito, assim teremos no mximo um primo. 5, 6, 7 no satisfazem o enunciado pelo mesmo raciocnio acima: 675, 765, 756 e 576 so compostos. 2, 7, 9 no satisfazem o enunciado pois todas as suas reordenaes so mltiplos de 9. Portanto, os dgitos que Maxi escolheu foram 1, 3, 6.
PROBLEMA 2

Dada uma circunferncia C e um ponto P exterior a ela, traam-se por P as duas tangentes circunferncia, sendo A e B os pontos de tangncia. Toma-se um ponto Q sobre o menor arco AB de C. Seja M a interseo da reta AQ com a perpendicular a AQ traada por P, e seja N a interseo da reta BQ com a perpendicular a BQ traada por P. Demonstre que, ao variar Q no arco AB, todas as retas MN passam por um mesmo ponto.
SOLUO DE LEANDRO FARIAS MAIA (FORTALEZA CE)

Como AP tangente a "C", ento: (QBA = (QAP = . Analogamente: (PBQ = . Agora veja que: i) PNDB inscritvel, pois (PNB = 90 = (PDB. Assim; (NPD = (NBD = e (NDP = (NBP = .

Sejam = (QBA, = (QAB e "D" um ponto sobre AB de modo que PD AB.

EUREKA! N21, 2005

Sociedade Brasileira de Matemtica

M X

N A

Q D

ii) PMDA inscritvel, pois (PMA = 90 = (PDA. Assim; (PDM = (PAM = e (MPD = (MAD = . Portanto, de i) e ii) temos que (NPD = (MDP = NP // MD. (NDP = (MPD = PM // ND. Logo, PMDN paralelogramo, e ento NM e PD se cruzam no ponto mdio (*), assim para qualquer "Q" e AB (menor), MN passar por um ponto fixo que o ponto mdio da altura PD. Prova de (*):
A B

M D

AB // DC (ACD = (CAB, BC // AD (ADB = (CBD.


EUREKA! N21, 2005

(ABD = (BDC.

Sociedade Brasileira de Matemtica

Assim, ACD ACB( LAAO ) DC = AB e AD = BC. MDC ABM ( ALA) AM = MC e BM = MD.


PROBLEMA 3

Seja n um inteiro positivo. Chamamos Cn a quantidade de inteiros positivos x, menores que 10n, tais que a soma dos dgitos de 2x menor que a soma dos dgitos de x. Demonstre que Cn 4 n (10 1) . 9

SOLUO:
j Se m = a j 10 , com j=0 k j a j {0,1, 2,...,9}, j k , temos 2m = (2a j ) 10 . Note j =0 k

que, se a {0,1, 2,3, 4}, 2a {0, 2, 4,6,8} tem apenas um dgito, e, se a {5,6,7,8,9}, 2a {10,12,14,16,18} tem dois dgitos, sendo o primeiro deles igual a 1. Assim, na soma
j

(2a ) 10
j j =0

, no h "vai um", pois, se (2ai ) 10i e

(2a j ) 10 tm dgitos no nulos na k-sima, posio, com i < j, ento k = j = i + 1, sendo o dgito de (2ai ) 10i igual a 1, nessa k-sima posio, donde sua soma menor que 10 (pois o dgito corresponde de (2a j ) 10 j no mximo 8). Portanto, se s(m) denota a soma dos dgitos de m, s (m) = a j e s (2m) = s (2a j ). Agora, para
j =0 j =0 k k

a {0,1, 2,3, 4}, s (2a ) = 2a, e, para a {5, 6, 7,8,9} , s (2a ) = 2a 9. Portanto, para 0 a 9, s (2a ) + s (2 (9 a )) = 2a + 2(9 a ) 9 = 9 = a + (9 a ). Assim, se x = a j 10 j , (com 0 a j 9, j ) um inteiro positivo menor que 10 n , temos
j=0 n 1

(10n 1) x = (9 a j ) 10 j , e logo, s (2 x ) + s (2(10n 1 x )) =


j =0

n 1

= s (2a j ) + s (2 (9 a j )) = ( s (2a j ) + s (2 (9 a j )) = (a j + (9 a j )) =
j =0 j =0 j =0 j =0

n 1

n 1

n 1

n 1

EUREKA! N21, 2005

Sociedade Brasileira de Matemtica

= a j + (9 a j ) = s ( x ) + s ((10n 1) x ). Assim, s (2 x ) < s ( x ) se e somente se


j =0 j =0

n 1

n 1

s (2(10 1 x )) > s (10n 1 x ), e s (2 x ) = s ( x ) se e somente se


n

10n 1 A , onde 2 A = #{1 x < 10n | s(2 x) = s( x)}. Note agora que s ( x ) x (mod 9) e s (2 x ) 2 x (mod 9) , donde, se s (2 x ) = s ( x ), ento 2 x x (mod 9) , e logo s (2(10n 1 x )) = s (10n 1 x ). Portanto, Cn = x 0(mod 9). Cn 4 n (10 1). 9 Assim, A #{1 x < 10 n | x 0(mod 9)} = 10n 1 , 9 donde

Arnaldo escolhe um inteiro a, a  H %HUQDOGR HVFROKH XP LQWHLUR b, b  $PERV dizem, em segredo, o nmero que escolheram a Cernaldo, e este escreve em um quadro os nmeros 5, 8 e 15, sendo um desses a soma a + b. Cernaldo toca uma campainha e Arnaldo e Bernaldo, individualmente, escrevem em papis distintos se sabem ou no qual dos nmeros no quadro a soma de a e b, e entregam seus papis para Cernaldo. Se em ambos os papis est escrito NO, Cernaldo toca novamente a campainha, e o procedimento se repete. Sabe-se que Arnaldo e Bernaldo so sinceros e inteligentes. Qual o nmero mximo de vezes que a campainha pode ser tocada at que um deles escreva que sabe o valor da soma?
SOLUO DE TELMO LUIS CORREA JNIOR (SANTO ANDR SP)

PROBLEMA 4

Se a 9, a campainha toca apenas 1 vez: Arnaldo sabe que b 0, logo a nica soma possvel 15. O mesmo ocorre se b 9, com Bernaldo. Caso contrrio, a campainha toca 2 vezes: com a, b 8 no possvel decidir entre as somas dadas na primeira vez. Se a = 6, Arnaldo agora sabe que b 8, ento a soma no pode ser 5 ou 15, apenas 8. O mesmo ocorre se b = 6. Se a {0,1, 2,3,4,5,7,8}, ainda no possvel decidir entre 5, 8 e 15. O mesmo ocorre para Bernaldo. Caso a campainha toque pela terceira vez a, b {0,1,2,3,4,5,7,8} .

EUREKA! N21, 2005

Sociedade Brasileira de Matemtica

Se a = 2, Arnaldo sabe que b 6 e b 8, logo a nica soma possvel 5. O mesmo ocorre se b = 2. Caso a 2 e b 2 , ningum escreve o SIM. De fato: Se a {0,1,3, 4,5}, Arnaldo no pode decidir entre as somas 5 e 8. Se a {7,8}, Arnaldo no pode decidir entre as somas 8 e 15. O mesmo ocorre para Bernaldo. A campainha toca pela quarta vez, e ambos sabem que a, b {0,1,3, 4,5,7,8}. Se a = 3, Arnaldo sabe que b 2, logo a soma deve ser 8. Se a {0,1, 4,5} no possvel decidir entre 5 e 8, se a {7,8} , no possvel decidir entre 8 e 15. O mesmo ocorre com Bernaldo. A campainha toca pela quinta vez: ambos sabem que a, b {0,1,4,5,7,8} . Se a = 5, Arnaldo sabe que b 3 , logo a soma deve ser 5. Se a {0,1, 4} no possvel decidir entre 5 e 8, se a {7,8} no possvel decidir entre 8 e 15. O mesmo para Bernaldo. A campainha toca: sexta vez - ambos sabem que a, b {0,1, 4,7,8}. Se a = 0, Arnaldo sabe que b 5 , logo a soma deve ser 8. Se a {1, 4}, no possvel decidir entre 5 e 8, se a {7,8}, entre 8 e 15. O mesmo para Bernaldo. Stima vez: ambos sabem que a, b {1,4,7,8}. Se a = 8, Arnaldo sabe que a soma deve ser 15, se a {1, 4}, no pode decidir entre 5 e 8, se a = 7, no pode decidir entre 8 e 15. O mesmo com Bernaldo. Oitava vez: ambos sabem que a, b {1,4,7}. Se a = 7, Arnaldo sabe que a soma deve ser 8, se a {1, 4}, no pode decidir entre 5 e 8. O mesmo para Bernaldo. Nona vez: ambos sabem que a, b {1,4}. Arnaldo sabe que nica soma possvel se a = 1 5; se a = 4, a soma pode ser 5 ou 8. O mesmo para Bernaldo. A campainha toca pela dcima vez: agora ambos sabem que a = b = 4, e determinam a soma com segurana. Logo, a campainha pode ser tocada no mximo dez vezes.
PROBLEMA 5

Utilizando triangulinhos eqilteros de papel, de lado 1, forma-se um tringulo eqiltero de lado 2 2004 . Desse tringulo retira-se o triangulinho de lado 1 cujo centro coincide com o centro do tringulo maior. Determine se possvel cobrir totalmente a superfcie restante, sem superposies nem buracos, dispondo-se somente de fichas em forma de trapzio issceles, cada uma formada por trs triangulinhos eqilteros de lado 1.
EUREKA! N21, 2005

Sociedade Brasileira de Matemtica

SOLUO DE GABRIEL TAVARES BUJOKAS (SO PAULO SP)

Vamos mostrar por induo que qualquer n 1 (mod 9) possvel cobrir o tringulo dividido em n partes cada lado. Como 22004 (+1) (mod 9) j que 26 n 1 e 6 | 2004, 26 n 1 (mod 9) e 6 | 2004 para n = 22004 possvel. Vamos mostrar que se para n 1 (mod 9) possvel, ento para n + 9 tambm . Temos que n + 9 1 (mod 9). Cobriremos os 3 lados do tringulo maior assim: Comece por um vrtice A e cubra um lado a que este pertena com peas at restarem 2 espaos (como n 1 (mod 3), isto possvel). Agora preencha da mesma forma a segunda linha, at restarem 4 espaos. Na terceira linha, deixe um espao esquerda e preencha at onde foi a segunda linha (como indicado na figura acima). E, acima das primeiras duas peas da terceira linha, ponha mais uma (como na figura). Faa isso para todos os vrtices. Observe que no h nem espaos nem superposies. Assim criamos uma "barba" e o meio um tringulo de lado (n + 9) 9 (muito fcil de ver, j que a borda tem "largura" 3). O centro do tringulo grande coincide com o do de lado n. Assim, por hiptese de induo, este tringulo central pode ser preenchido, logo o tringulo grande tambm!. A base da induo n =1, caso em que no h nada a fazer. Assim, nossa afirmao est provada.

EUREKA! N21, 2005

Sociedade Brasileira de Matemtica

PROBLEMA 6

Sejam m, n inteiros positivos. Em um tabuleiro m n, quadriculado em quadradinhos de lado 1, considere todos os caminhos que vo do vrtice superior direito ao inferior esquerdo, percorrendo as linhas do quadriculado exclusivamente nas direes e . Define-se a rea de um caminho como sendo a quantidade de quadradinhos do tabuleiro que h abaixo desse caminho. Seja p um primo tal que rp(m) + rp(n) p, onde rp(m) representa o resto da diviso de m por p e rp(n) representa o resto da diviso de n por p. Em quantos caminhos a rea um mltiplo de p?
SOLUO:

Para resolver este problema, usaremos tcnicas de funes geratrizes descritas no artigo "Sries formais", de Eduardo Tengan, da Eureka! No. 11, pp. 34 39. Representaremos o tabuleiro por {1, 2,,m} {1, 2,,n}. Desta forma, os quadradinhos so representados por pares (i, j), com 1 i m,1 j n. Considere agora um caminho como no enunciado. Para 1 i m, seja bi o nmero de quadradinhos da forma (i, y) que esto abaixo do caminho. Como o caminho s desce ou anda para a esquerda, temos 0 b1 b2 b3 ... bm (e, reciprocamente, tais bi determinam um caminho). A rea do caminho yi = bi bi 1 . k m + 1,
m

b .
i =1 i

Definamos bm +1 = n e b0 = 0. Para 1 i m + 1, seja

Temos
i

y
i =1 m+1 i =1

m +1

= bn +1 b0 = n , Portanto,
m

e, rea

em do

geral, caminho

para

y
i =1 m i =1

= bk b0 = bk .

A = bi = (m + 1 i) yi = (m + 1 i) yi = j ym+1 j .
i =1 j =0

Assim, fazendo z j = ym +1 j , temos

zj = n e
j =0

jz
j =0

= A.
m

Assim, o problema equivalente a achar o nmero de solues de z0 + z1 + ... + zm = n, com zi 0 para 0 i m + 1 e isso, considere a funo F ( x, y ) =
EUREKA! N21, 2005
m

jz
j =0

mltiplo de p. Para

1 1 1 1 . Como = ... k 1 y 1 xy 1 xm y k =0 1 x y

10

Sociedade Brasileira de Matemtica

1 = 1 + x k y + ( x k y ) 2 + ..., F ( x, y ) = ar , s x r y s , onde ar , s o nmero de k 1 x y r , s< solues de z0 + z1 + ... + zm = s com zi 0, i 0 e zi + 2 z2 + 3 z3 + ... = jz j = r.


j =0 m

Em particular, ar ,n o nmero de caminhos como no enunciado com rea r. Queremos ento calcular

a
p|r

r ,n

, i.e., a soma dos ar ,n para os r mltiplos de p.

Para isso, consideremos a soma unidade (e logo

F (
k =0

p 1

, y ), onde = e 2 i

uma raiz p-sima da

k =0

p 1

kr

= 0, sempre que r no mltiplo de p). Temos kr y s =

F (
k =0

p 1

, y) =
k =0

p 1

r , s

< a

r ,s

r , s

< a

r ,s

p 1 kr y s = p ar ,s y s (pois, se r , s< k =0
p|r

p | r,

kr = p ) . Em particular, o coeficiente de y n em
k =0

p 1

F (
k =0

p 1

, y ) p vezes o

nmero de caminhos como no enunciado cuja rea um mltiplo de p. 1 = (1 + y + y 2 + ...)m +1 , e o coeficiente de y n m +1 (1 y ) em F (1, y ) o nmero de solues de k1 + k2 + ... + km +1 = n , ki 0, i , que
m m+n 1 1 m 1 = m+1 1 kr . . Por outro lado, para 1 k p 1, F ( k , y ) = kr y r =0 y r =0 1 y m Escrevendo m = qp + rp ( m) = ( q + 1) p ( p rp (m)), com 0 rp (m) < p, temos ( q +1) p 1

Para k = 0, F ( k , y ) = F (1, y ) =

1 r ( q +=mp+11 F ( , y ) = 1) y (q + 1) p
k

(1 y )
kr

( q +1) p 1

r =0

(y
1 kr

kr

1 y (q +1) p

r =m +1 p

(y

(1 kr y ) 1) q +1 =

( q +1) p 1 r =m +1

(1 y )
kr

(1 y )

+ p q 1

( q +1) p 1

Como o produto

r = m+1

(1 y )

tem p rp (m) 1 termos, ao desenvolver esse

produto, obtemos termos da forma a y s , com 0 s p rp ( m) 1. Por outro lado, todas as potncias de y em
EUREKA! N21, 2005
q +1 1 = (1 + y p + y 2 p + ...) tm expoente mltiplo p q +1 (1 y )

11

Sociedade Brasileira de Matemtica

de p, donde os termos no-nulos em F ( k , y ) tm o expoente de y congruente a s mdulo p, para algum s com 0 s p rp ( m) 1. Entretanto, rp ( m) + rp ( n) p, donde rp ( n) > p rp ( m) 1 , e, como n congruente a rp ( n) mdulo p, nenhum termo no nulo em F ( k , y ) tem expoente n. Assim, o nmero procurado de caminhos cuja rea um mltiplo de p 1 m + n . p m

4XH 225964951 1 p SULPR" (VWH Q~PHUR GH  GtJLWRV p R PDLRU SULPR FRQKHFLGR H p XP SULPR GH 0HUVHQQH DVVLP FRPR R VHJXQGR R WHUFHLUR H R TXDUWR PDLRUHV SULPRV FRQKHFLGRV TXH VmR 2 24036583 1, 220996011 1 H 213466917 1. 2 GHVFREULGRU GHVWH QRYR SULPR R   SULPR GH 0HUVHQQH FRQKHFLGR  'U 0DUWLQ 1RZDN p XP RIWDOPRORJLVWD DOHPmR TXH SDUWLFLSD GR *,036 XP SURMHWR FRRSHUDWLYR SDUD SURFXUDU SULPRV GH 0HUVHQQH UHVSRQViYHO SHOD GHVFREHUWD GRV RLWR PDLRUHV SULPRV GH 0HUVHQQH DWXDOPHQWH FRQKHFLGRV 9HMD KWWSZZZPHUVHQQHRUJ SDUD PDLRUHV LQIRUPDo}HV
4

9RFr VDELD

EUREKA! N21, 2005

12

Sociedade Brasileira de Matemtica

XLV OLIMPADA INTERNACIONAL DE MATEMTICA


Enunciados e Solues
Caros leitores: nossa equipe da IMO-2004 tinha seis alunos muito diferentes, mas com uma caracterstica em comum: garra. Todos eles adotaram a ttica de escrever tudo o que passava pelas suas cabeas, de modo a maximizar as suas pontuaes. Assim, no podemos transcrever as suas solues ipsis literis. Ao contrrio, vamos colocar somente partes das solues deles, com alguns comentrios sobre as suas idias. Carlos Yuzo Shine, vice-lder do Brasil na IMO-2004 PRIMEIRO DIA
PROBLEMA 1

Seja ABC um tringulo acutngulo com AB AC. A circunferncia de dimetro BC intersecta os lados AB e AC nos pontos M e N, respectivamente. Seja O o ponto mdio do lado BC. As bissetrizes dos ngulos BAC e MON intersectam-se em R. Prove que as circunferncias circunscritas aos tringulos BMR e CNR tm um ponto em comum que pertence ao lado BC.

n
C

SOLUO DE THIAGO COSTA LEITE SANTOS (SO PAULO - SP)


A

/2 /2
M B

/ 2 / 2

2
K O

EUREKA! N21, 2005

13

Sociedade Brasileira de Matemtica

Seja a circunferncia com dimetro BC, B o circuncrculo de BMR e C o circuncrculo de CNR. Suponhamos sem perda de generalidade que o circunraio de ABC mede 1/2, de modo que BC = sen , AC = sen e AB = sen . Aqui, Thiago nota (e prova) que o ponto comum s duas circunferncias e o lado BC s pode ser o p da bissetriz: Temos pot A = AM AB = AN AC. Mas pot B A = AM AB e potC A = AN AC, logo A pertence ao eixo radical de B e C. Como R tambm pertence ao eixo radical, a reta AR o eixo radical de B e C. Logo o ponto comum a B e C e BC s pode ser a interseo de AR e BC, ou seja, a interseo da bissetriz de BAC e BC. Mais algumas consideraes geomtricas: como OM = ON (raios de ), NOR = MOR (OR bissetriz de MON) e OR comum, os tringulos MOR e NOR so congruentes (caso LAL), portanto RNO = RMO. Nesse momento, Thiago percebe como poderia terminar o problema: seja K a interseo da bissetriz de BAC e BC. A idia dele provar que K pertence a BC, B e C. Que pertence a BC bvio. Para provar que pertence a B, basta provar que B, M, R e K so concclicos; e, de modo anlogo, para provar que pertence a C, basta provar que C, N, R e K so concclicos. Aqui est a formalizao dele: se provarmos que RNO = /2, teramos RNC = + /2 e RNC + RKC = + /2 + + /2 = , portanto K pertence a C. Analogamente (aqui voc deve verificar que Thiago usa o fato que ele provou antes de que RNO = RMO), vamos ter K pertencente a B, portanto B e C tm um ponto comum em BC. Deste modo, s falta provar que RNO = /2. Isso um problema que costuma ser resolvido de duas maneiras: (1) com arrasto (isto , calculando todos os ngulos na figura) ou (2) arrasto seguido de contas (isto , com trigonometria, geometria analtica ou complexos). Thiago resolveu adotar a opo (2), com trigonometria (quando o coordenador viu as contas, ele comentou, estava indo to bem at aqui, por que ele teve que fazer essas contas??)

EUREKA! N21, 2005

14

Sociedade Brasileira de Matemtica

O tringulo NBO issceles, logo ONC = OCN = e NOC = 2. ( 2) ( 2 ) = Analogamente, MOB = 2 e = MOR = NOR = 2 . 2 Seja RNO = . A idia calcular alguma funo trigonomtrica de (geralmente acaba sendo a cotangente, por causa do truque da cotangente veja o artigo Geometria com Contas, Eureka! 17) e compar-la com a mesma funo trigonomtrica de /2 para provar que = /2. Mas por onde comear? Primeiro devemos procurar por tringulos que envolvam . Um deles vem da prpria definio de : RNO. Outro bem interessante o ARN, cujos ngulos envolvem tanto como /2, alm de ter RN em comum (um segmento a menos para calcular, como voc vai ver depois!). Ento parece ser uma boa estratgia utilizar esses dois tringulos. Lei dos senos em RNO: RN NO = sen ( 2 ) sen ( 2 + ) Lei dos senos em ARN: RN AN = sen 2 sen ( + ) 2 (1)

(2)

Veja que se dividirmos (1) por (2), RN se cancela, ento no precisamos calcular BC sen . Alm disso, = RN. S precisamos de NO e AN. NO fcil: o raio 2 2 AN = AC NC = sen NC. S falta NC! Pela lei dos senos em NOC, NC OC sensen2 sen 2sen cos = NC = = = sen cos 2 sen 2 sen sen2 sen logo AN = sen sen cos = sen( + ) sen cos = sen cos + sen cos sen cos = sen cos . Vamos usar repetidas vezes o fato de que sen(/2 x) = cos x e as frmulas de Prostafrese (que os gregos tiveram que deduzir na coordenao! procure-as na
EUREKA! N21, 2005

15

Sociedade Brasileira de Matemtica

Eureka! 17). Agora sim, dividindo (1) por (2) e substituindo NO e AN, obtemos a nossa conta final:
sen sen ( + ) sen sen 2 sen ( + 2 ) 2 2 = = cos sen cos cos( ) cos( ) cos 2

sen cos( ) = sen ( + ) cos (*) 2 2 Parece loucura, mas Thiago abriu os senos e co-senos. Mas isso tinha um motivo: ele queria achar cotg (lembre-se: o segredo do sucesso nas contas sempre ter uma meta em mente!).
(*) cos cos sen + sensen sen = sen cos ( ) cos + cos sen ( ) cos 2 2 2 2 cos cos sen sen ( ) cos = sen cos ( ) cos sensen 2 2 2 2 cos ( sen( + ) + sen( ) sen sen( )) =

sen(cos + cos( ) cos( ) + cos( + ))


+ cos cos 2 sen ( 2 ) sen ( 2 ) sen ( ) 2 = = = cotg cotg = + 2 sen sen 2sen ( 2 ) cos ( 2 ) cos ( 2 )

Como cotg = cotg que precisvamos.

e e pertencem ao intervalo ]0; [, temos = ,o 2 2 2

PROBLEMA 2

Determine todos os polinmios P(x) de coeficientes reais que satisfazem a igualdade


P( a b) + P(b c) + P(c a ) = 2 P( a + b + c )

para quaisquer nmeros reais a, b, c, tais que ab + bc + ca = 0.


SOLUO DE FBIO DIAS MOREIRA (RIO DE JANEIRO - RJ)

Note inicialmente que P(a b) + P(b c) + P(c a) = 2P(a + b + c) P(b a) + P(a c) + P(c b) = 2P(b + a + c) (*) P(b a) + P(c b) + P(a c) = 2P( a b c) (**)
EUREKA! N21, 2005

16

Sociedade Brasileira de Matemtica

Desde que ab + bc + ca = 0 (a primeira equao uma aplicao direta; a segunda segue se permutarmos a e b; a terceira se levarmos (a; b; c) em ( a; b; c), o que permitido pois ( a)( b) + ( b)( c) + ( c)( a) = ab + bc + ca = 0). Igualando (*) e (**), P(a + b + c) = P( a b c) desde que ab + bc + ca = 0. Mas dado S real, se tomarmos a = S, b = 0 e c = 0, ento a + b + c = S e ab + bc + ca = 0. Logo P(x) = P(x) para todo x real, ou seja, P par. Em particular, isto implica que o grau de P, gr(P), par. Agora vem a parte da soluo que usa anlise. Ele provou um lema geral bem til para o problema. Note inicialmente que se P(x) = k para todo x real ento 3k = 2k k = 0 que, de fato, uma soluo. Suponha agora que P no constante, de grau n. Seja Qx() = P(x) (aqui, Fbio usa uma idia relacionada a homogeneidade). Fixado x, Qx um polinmio em de grau n (de fato, de grau n se x 0 e de grau 0 se x = 0). Lema. lim Qx () = Cn x n , sendo Cn o coeficiente lder de P(x). n

Demonstrao: Note que Qx(), Qx(), , Qx(n 1)(), n, n 1, , so todos polinmios, logo so derivveis, e que para um desses polinmios qualquer, digamos R, temos lim | R() | = + , pois todos eles tm grau maior ou igual a 1. Ento, aplicando o

Qx () Q ( n ) () . Mas se P(x) = lim x + n + n! = Cnxn + + C1x + C0, Qx() = Cnxnn + + C1x + C0, logo Qx(n)() = n!Cnxn, logo o limite desejado mesmo Cnxn. teorema de LHpital repetidas vezes, temos lim Agora, Fbio aproveita-se da homogeneidade da condio ab + bc + ca = 0: como ab + bc + ca = 0 (a)(b) + (b)(c) + (c)(a) = 0 para todo real, logo P((a b)) + P((b c)) + P((c a)) = 2P((a + b + c)) Qab() + Qbc() + Qca() = 2Qa+b+c() para todos , a, b, c reais, ab + bc + ca = 0. Q () Como cada um dos limites lim x n , x {a b; b c; c a}, n = gr(P) existe, + existe o limite da soma, que a soma dos limites. Logo

EUREKA! N21, 2005

17

Sociedade Brasileira de Matemtica

( ) Qa b () + Qb c ( ) + Qc a () Q = 2 lim a +b +nc n n n n Cn (a b) + Cn (b c ) + Cn (c a ) = 2Cn (a + b + c )n sendo a, b, c reais tais que ab + bc + ca = 0 e n = gr(P). Como Cn, coeficiente lder de P, no-nulo por hiptese, (a b)n + (b c)n + (c a)n = 2(a + b + c)n (***) Tome a = 2, b = 3 e c = 6 (note que ab + bc + ca = 0). Ento, substituindo em (***), (5)n + (3)n + 8n = 2 7n Como n par, obtemos 5n + 3n + 8n = 2 7n. Se n 7,

lim

1 8 1 8n = 7 n 7 n 1 + > 7 n 1 + 7 = 2 7 n , absurdo. 7 7 7 Logo n 6. Mas n = 6 implica 0 2 76 36 + 56 + 86 1 + 1 + 1 3 (mod. 7), absurdo. Logo P(x) = Ax2 + B ou P(x) = Ax4 + Bx2 + C. Fbio no precisava estudar o caso P(x) = Ax2 + B, pois isso um caso particular do caso P(x) = Ax4 + Bx2 + C, mas ele estudou. Na hora de estudar o caso P(x) = Ax4 + Bx2 + C, ele viu que a conta seria grande. Mas nesse momento ele lembrou as palavras que o prof. Luciano sempre diz: tem que ter garra para ser campeo! e no fraquejou, fazendo as contas: Substitua a = b = c = 0 na equao do enunciado. Segue imediatamente que C = 0. Logo P(x) = Ax4 + Bx2. Substituindo (isso mesmo!) e tendo em mente que ab + bc + ca = 0 e que hay que hacer las cuentas, pero sin perder la simetra jams, (a b)2[A(a b)2 + B] + (b c)2[A(b c)2 + B] + (c a)2[A(c a)2 + B] = = 2(a + b + c)2 [A(a + b + c)2 + B] A[(a b)4 + (b c)4 + (c a)4] + B[(a b)2 + (b c)2 + (c a)2] = 2A(a + b + c)4 + 2B(a + b + c)2 A[a4 4a3b + 6a2b2 4ab3 + b4 + b4 4b3c + 6b2c2 4bc3 + c4 + c4 4c3a + 6c2a2 4ca3 + a4] + B[a2 2ab + b2 + b2 2bc + c2 + c2 2ca + a2] = 2A(a2 + b2 + c2 + 2ab + 2bc + 2ca)2 + 2B(a2 + b2 + c2 + 2ab + 2bc + 2ca) 2A[a4 + b4 + c4 2ab(a2 + b2) 2bc(b2 + c2) 2ca(c2 + a2) + 3a2b2 + 3b2c2 + 3c2a2] + 2B[a2 + b2 + c2] = 2A[a4 + b4 + c4 + 2a2b2 + 2b2c2 + 2c2a2] + 2B[a2 + b2 + c2] A[a2b2 + b2c2 + c2a2 2ab(a2 + b2) 2bc(b2 + c2) 2ca(c2 + a2)] = 0 A[a2b2 + b2c2 + c2a2 2a2(ab + bc + ca) + 2a2bc 2b2(ab + bc + ca) + 2ab2c 2c2(ab + bc + ca) + 2abc2] = 0 A[(ab + bc + ca)2 2abc(a + b + c) + 2abc(a + b + c)] = 0 0 = 0.
EUREKA! N21, 2005

18

Sociedade Brasileira de Matemtica

Como todas as passagens so equivalncias, no h restrio sobre A e B. Portanto os nicos polinmios que satisfazem as condies do enunciado so os da forma P(x) = Ax4 + Bx2, A, B reais.
PROBLEMA 3

Um gancho uma figura formada por seis quadrados unitrios como no seguinte diagrama

ou qualquer uma das figuras obtidas desta aplicando rotaes ou reflexes. Determine todos os retngulos m n que podem ser cobertos com ganchos de modo que: i) O retngulo coberto sem buracos e sem sobreposies; ii) Nenhuma parte de nenhum gancho pode cobrir regies fora do retngulo.

SOLUO OFICIAL

Considere um preenchimento do tabuleiro m n. Para cada gancho A, existe um nico gancho B cobrindo o quadradinho interno de A com uma de suas extremidades. Alm disso, o quadradinho interno de B deve ser coberto por uma das extremidades de A. Assim, num recobrimento, todos os ganchos formam pares. H apenas duas maneiras de formar tais pares. Em um caso, A e B formam um retngulo 3 4; no outro, a sua unio um octgono, com lados 3, 2, 1, 2, 3, 2, 1, 2. Ento um tabuleiro m n pode ser coberto por ganchos se e somente se pode ser coberto com os pares de 12 quadradinhos acima. Suponha que tal recobrimento exista. Ento mn divisvel por 12. Provaremos que m ou n divisvel por 4. Assuma por absurdo que isso no acontece; ento m e n so ambos pares, pois mn divisvel por 4. Imagine o tabuleiro dividido em quadradinhos unitrios, com linhas e colunas rotuladas 1, 2, , m e 1, 2, , n, respectivamente. Escreva 1 no quadradinho (i, j) se exatamente um entre os nmeros i e j divisvel por 4, e 2, se i e j so ambos divisveis por 4. Como o nmero de quadrados em cada linha e coluna par, a soma de todos os nmeros escritos par. Mas no difcil verificar que um retngulo 3 4 sempre cobre nmeros com soma 3 ou 7; e o outro tipo de par sempre cobre nmeros com soma 5 ou 7. Conseqentemente, o nmero de pares de peas par. Assim, mn divisvel por 24 e, portanto, por 8, absurdo, pois supusemos que nem m nem n mltiplo de 4.

EUREKA! N21, 2005

19

Sociedade Brasileira de Matemtica

Note tambm que nem m nem n pode ser 1, 2 ou 5 (qualquer tentativa de colocar as peas nesses casos falha). Conclumos ento que se um recobrimento possvel ento m ou n divisvel por 3, m ou n divisvel por 4 e m, n {1; 2; 5}. Reciprocamente, se essas condies acima so verificadas, o recobrimento possvel (utilizando somente retngulos 3 4). Isso imediato quando 3 divide m e 4 divide n (ou vice-versa). Seja m divisvel por 12 e n {1; 2; 5} (ou vice-versa). Ento n pode ser representado como a soma de vrios 3s e vrios 4s. Ento o tabuleiro pode ser particionado em retngulos m 3 e m 4, que so fceis de cobrir, utilizando novamente retngulos 3 4. SEGUNDO DIA
PROBLEMA 4 Seja n 3 um inteiro. Sejam t1 , t2 ,..., tn nmeros reais positivos tais que

1 1 1 n 2 + 1 > (t1 + t2 + ... + tn ) + + ... + . tn t1 t2

Mostre que ti , t j e tk so as medidas dos lados de um tringulo para quaisquer i, j, k com 1 i < j < k n.
SOLUO DE HENRY WEI CHENG HSU (SO PAULO - SP)

Suponha tn tn 1 t2 t1 e seja tn = t1 + t2 + k. Temos 1 1 1 1 1 t1 + t2 + "+ tn1 1 + tn + + " + n2 + 1 > (t1 + t2 + "+ tn1 ) + + "+ + +1 tn1 tn tn1 t1 t2 t1 t2

Por Chebyshev (na verdade, poderamos usar Cauchy ou MA-MH), 1 1 1 + +" + tn 1 n 1 (t1 + t2 + " + tn 1 ) t1 t2 n 1 n 1 n 1 1 1 1 2 (t1 + t2 + " + tn 1 ) + + " + (n 1) t1 t2 tn 1 Assim,
EUREKA! N21, 2005

20

Sociedade Brasileira de Matemtica

1 1 1 t1 + t2 + " + tn 1 + tn + + " + tn tn 1 t1 t2 t +t t t t + " + tn 1 tn t 2n 1 > 1 2 + n + n + 3 + +"+ n tn t1 t2 tn t3 tn 1 t t Como k + n 2 , temos tn tk t +t t t t +t t t 2n 1 > 2n 6 + 1 2 + n + n 5 > 1 2 + n + n tn t1 t2 tn t1 t2 Sendo tn = t1 + t2 + k, k t +t t + t + k t1 + t2 + k k t k t + 5 > 1 +1+ 2 + + 1 +1+ 5> 1 2 + 1 2 + t2 + k + t2 + k t1 t1 t2 t1 t1 t2 t2 t1 n2 + 1 > (n 1)2 + 1 + t1 t2 k k k + + + t2 t1 t2 t1 t1 + t2 + k t t Mas 1 + 2 2 . Deste modo, t2 t1 2> 1 1 1 1 1 1 0> k + 0 > k + t1 t2 t1 + t2 + k t1 t2 tn 1 1 1 Veja que + > pois t1 t2 tn. Logo k < 0 e tn < t1 + t2. t1 t2 tn O resto mais fcil: sejam 1 i < j n. Temos ti + tj t1 + t2 > tn tk para 1 k n e, portanto, ti, tj e tk so sempre lados de um tringulo.
PROBLEMA 5

Num quadriltero convexo ABCD a diagonal BD no bissetriz do ngulo ABC nem do ngulo CDA . Um ponto P no interior de ABCD satisfaz PBC = DBA e PDC = BDA.

Prove que os vrtices do quadriltero ABCD pertencem a uma mesma circunferncia se e s se AP = CP.

EUREKA! N21, 2005

21

Sociedade Brasileira de Matemtica

SOLUO DE RAFAEL DAIGO HIRAMA (CAMPINAS - SP)

1 1 2

A P

2 1
D

Primeiro ato: consideraes gerais. Vamos determinar a figura a partir de quatro ngulos: sejam ABD = 1, BDA = 1, CBD = 2, CDB = 2. Ento DBP = 2 1 e BDP = 2 1. Veja que esses ngulos no dependem de P estar dentro do tringulo ABD (o que implica ABD > DBC e ADB > BDC) ou P estar dentro do tringulo BCD (o que implica ABD < DBC e ADB < BDC) pois se ABD = PBC ento ABD DBP = PBC DBP (+ se P est dentro de BDC e se P est dentro de ABD) ABP = DBC = 2. Para os ngulos com vrtice D anlogo. Usando lei dos senos nos tringulos ABP, BCP, ADP e DCP, temos AP AB CP BC = = (1) (2) sen 2 senAPB sen1 senBPC AP AD CP DC = = (3) (4) sen2 senAPD sen1 senDPC De (1) e (4), temos AP AB sen 2 senDPC = (I) CP DC sen1 senAPB

EUREKA! N21, 2005

22

Sociedade Brasileira de Matemtica

De (2) e (3), temos AP AD sen2 senBPC (II) = CP BC sen1 senAPD As equaes (I) e (II) so fundamentais para a soluo do problema, pois envolvem AP e CP, lados do quadriltero ABCD e, alm disso, observando a figura e essas equaes, notamos algumas simetrias interessantes. Rafael comea a us-las a partir de agora. Vamos l: senAPD = sen(APD + BPC BPC) = sen(APD + BPC) cosBPC cos(APD + BPC) senBPC. Seja = APD + BPC = 360 (APB + CPD). Logo senAPD = sen cosBPC cos senBPC (III) e senAPB = sen(APB + CPD CPD) = sen(APB + CPD) cosCPD sen(APB + CPD) cosCPD = sen cosCPD cos senCPD (IV) (lembre-se de que sen(360 x) = sen x e cos(360 x) = cos x). Segundo ato: ABCD cclico AP = CP. Supondo que ABCD cclico (esta a primeira vez que vamos usar este fato), AB CD AD BC temos, pela lei dos senos estendida, que . e = = sen1 sen2 sen1 sen 2 Substituindo essas duas ltimas igualdades e (III) e (IV) em (I) e (II), respectivamente, temos PC senAPD sen (VI): = = cos AP senBPC tg BPC PC senAPB sen (V): = = cos AP senCPD tg CPD Logo tgBPC = tg CPD ou sen = 0. A primeira possibilidade implica BPC + CPD = 180, mas a B, P e C seriam colineares e ento BD bissectaria os ngulos ABC e/ou ADC. Portanto sen = 0 e ento = 180, pois = APD + BPC est entre 0 e 360. Ento PC sen180, = cos180, = 1, ou seja, AP = PC. AP tg BPC

EUREKA! N21, 2005

23

Sociedade Brasileira de Matemtica

Terceiro ato: AP = CP ABCD cclico. AD sen2 senAPD CD sen1 senCPD e . = = AB sen 2 senAPB BC sen1 senBPC AD sen2 CD sen1 senAPD senCPD Multiplicando, temos . = BC sen1 AB sen 2 senBPC senAPB De (II) e (I), temos Aplicando a lei dos senos em ABD e BCD temos Assim, senAPD senAPB = senBPC senCPD AD AB CD BC e . = = sen1 sen1 sen 2 sen 2

Mas, fazendo as mesmas contas que fizemos em (V) e em (VI) (veja que as igualdades direita em (V) e (VI) no dependem de ABCD ser cclico), vemos que senAPD senAPB AD sen2 AD BC = 1. E ento . Sejam R1 = = =1 BC sen1 sen1 sen 2 senBPC senCPD e R2 os circunraios de ABD e BCD. Pela lei dos senos estendida, AD BC 2 R1 = = = 2 R2 R1 = R2. sen1 sen2 Logo os tringulos ABD e BCD tm o mesmo circunraio. S temos duas possibilidades para um circuncrculo de raio fixado em um segmento fixado: uma das possibilidades A, B, C e D serem concclicos, que o que queremos; a outra a circunferncia que passa por A, B e D ser a simtrica da que passa por B, C e D em relao a BD.
B

1
D
EUREKA! N21, 2005

24

Sociedade Brasileira de Matemtica

Neste ltimo caso vamos ter BAD = BCD. Como 1 + 1 + BAD = 180 e 2 + 2 + BCD = 180, 1 + 1 = 2 + 2. Mas veja que (1 > 2 e 1 > 2) ou (1 < 2 e 1 < 2) pois caso contrrio, como P est no interior do quadriltero ABCD, P deveria estar no interior de ABD e BCD ao mesmo tempo, o que impossvel. Logo no possvel que 1 + 1 = 2 + 2, e o problema acabou. Enfim, uma soluo muito parecida com uma trilogia grega, porm sem nenhuma tragdia.
PROBLEMA 6

Um inteiro positivo dito alternante se, na sua representao decimal, quaisquer dois dgitos consecutivos tm paridade diferente. Determine todos os inteiros positivos n tais que n tem um mltiplo que alternante.
SOLUO DE RAFAEL MARINI SILVA (RIO DE JANEIRO - RJ)

O principal diferencial da soluo do Marini que, em vez de procurar exemplos particulares para mltiplos alternantes de n, como a maioria dos estudantes fez, ele deu uma caracterizao bastante interessante dos nmeros alternantes. Vamos soluo: Seja T o conjunto dos nmeros cuja representao decimal consiste em algarismos todos menores do que 5. Veja que todo nmero alternante a pode ser escrito em uma das duas seguintes formas: se a mpar, a = 101010...101 + 2k = 

2 n1 dgitos

102 n 1 + 2k, k T, k com no mximo 2n 99

dgitos; (I) se a par, a = 101010...1010 + 2k = 

2 n dgitos

102 n 1 10 + 2k, k T, k com no mximo 99

2n + 1 dgitos; (II) Agora, Marini divide o problema em casos:


Caso 1: mdc(10; p) = 1.

Aqui, Marini toma sim um nmero particular. No muito difcil encontrar um nmero alternante nesse caso, se voc conhece o teorema de Euler-Fermat.

EUREKA! N21, 2005

25

Sociedade Brasileira de Matemtica

Tome k = 0 na forma (I) e n = (99p). Pelo teorema de Euler-Fermat, 10 (99p) 1 102 (99 p ) 1 (mod 99p) (veja que mdc(10; 99p) = 1) e portanto o nmero alternante 99 mltiplo de p.
Caso 2: p par, mas no mltiplo de 5.

Assim, p = 2e+1u, sendo u mpar e no mltiplo de 5. O nmero alternante que 102 n 1 10 + 2k, devemos encontrar da forma (II). Assim, 2e+1u| 99 102 n 1 5 + k, k T, k com no mximo 2n + 1 algarismos. 2eu| 99 102 n -1 5 + k. Para A nova dificuldade nesse caso a potncia de 2, ou seja, que 2e| 99 102 n 1 5 + k 0 (mod 2e) n suficientemente grande (maior que e/2), temos 99 k 5 991 (mod 2e). Marini resolveu, ento, provar um lema um pouco mais geral, mas que faz muito sentido: o conjunto T contm uma boa parte dos nmeros inteiros, ento
m Lema. Fixados a e m naturais e q {2,5} , existe t T tal que t a (mod q ).

Demonstrao. Induo sobre m.

Base de induo: m = 1: imediato, pois T > {0, 1, 2, 3, 4} > {0, 1}. Passo de induo: suponha o fato vlido para m = k. Vamos provar para m = k + 1. Suponha que 0 t < 10k . Seja Xt = {t; t + 10k; ; t + (q 1) 10k}. Note que esse conjunto est contido em T (pois q 5) e que Xt contm todos os possveis restos congruentes a t mdulo qk+1 (qk+1 no divide 10k e q divide 10). Da hiptese de induo, dado x natural existe t T com t T com t < 10k (se t 10k , trocamos t pelo nmero formado pelos seus k ltimos dgitos que congruente a t mdulo 10k, e logo mdulo qk) tal que x t (mod qk). Vendo mdulo qk+1 temos que x congruente a um elemento de Xt. Considerando a unio de todos os conjuntos Xt, com t variando entre 0 e qk 1, vemos que cobrimos todos os possveis restos (de fato, como Xt tem q elementos, a unio tem qk+1 elementos) e portanto o lema est demonstrado.
EUREKA! N21, 2005

26

Sociedade Brasileira de Matemtica

Assim, aplicando o lema congruncia k 5 991 (mod 2e), vemos que podemos escolher k0 T que satisfaz essa congruncia. Seja m o nmero de dgitos de k0. Tome s cpias de k0 (com alguns zeros esquerda para que fique com m + e + 1 (10m +e +1 )s 1 s 1 ( m + e +1) j . Veja que dgitos) e concatene-as, isto , seja k = k0 10 = k0 10m +e +1 1 j =0 k ainda um elemento de T e que k 5 991 (mod 2e), pois 10m+e+1 0 (mod 2e). Vamos fazer com que k seja mltiplo de u. Para isso, basta tomar s = ((10m+e+1 1) u) que o resto segue de Euler-Fermat. 102 n 1 10 + 2k. J 99 encontramos k. Agora, basta escolher um n adequado par acertar o nmero de algarismos de a. Tome n = (99u)v maior que a quantidade de dgitos de k. Pronto! Temos 102 n 1 102 n 1 10 + 2k = 2( 5 + k) 0 (mod 2e+1) (graas ao lema!) a= 99 99 102 n 1 10 + 2k 0 + 0 0 (mod u) a= 99 Logo a mltiplo de p. Retomando: o nmero alternante que procuramos a =
Caso 3: p mltiplo de 5, mas mpar. Seja p = 5eu, u mpar. Agora vamos procurar um nmero alternante da forma (I), ou 102 n 1 + 2k, k T. Para n suficientemente grande, seja, da forma 99 102 n 1 + 2k 0 (mod 5e) k 1981 (mod. 5e) 99

Novamente pelo lema, existe k0 T que satisfaz essa congruncia. Assim como no 10m +e +1 )s 1 ( , com = k0 caso anterior, tomemos k = k0 10 10m +e +1 1 j =0 m+e+1 1) u) e n = (99u)v maior que a quantidade de dgitos de k. De s = ((10 modo anlogo ao caso anterior, conseguimos um alternante mltiplo de p.
s 1 m + e +1

Caso 4: p par e mltiplo de 5, ou seja, p mltiplo de 10.


EUREKA! N21, 2005

27

Sociedade Brasileira de Matemtica

Neste caso, devemos tomar um alternante da forma (II), ou seja, a = 2k, k T. Seja p = 10u. Temos que 10u|

102 n 1 10 + 99

102 n 1 10 + 2k 5|k. Como k T, seu 99 ltimo dgito s pode ser zero, de modo que 10|k. Seja k = 10w, w T. Temos ento 102 n 1 102 n 1 + 2w, w T. Como + 2w mpar, u deve ser mpar, e que u| 99 99 camos no caso 1 ou 3. Logo existe um mltiplo alternante de u s quando u mpar, ou seja, quando p no mltiplo de 20. Logo os inteiros positivos que no possuem mltiplo alternante so exatamente os mltiplos de 20.

9RFr VDELD

4XH H[LVWHP SURJUHVV}HV DULWPpWLFDV DUELWUDULDPHQWH ORQJDV IRUPDGDV DSHQDV SRU Q~PHURV SULPRV" ,VWR IRL SURYDGR UHFHQWHPHQWH SRU %HQ *UHHQ H 7HUHQFH 7DR GRLV H[ROtPSLFRV  H HUD DWp HQWmR XPD FRQMHFWXUD EDVWDQWH DQWLJD  R ~OWLPR SURJUHVVR LPSRUWDQWH HP VXD GLUHomR KDYLD VLGR R UHVXOWDGR GH 9DQ GHU &RUSXW TXH SURYRX HP  TXH H[LVWHP LQILQLWDV SURJUHVV}HV DULWPpWLFDV GH WUrV WHUPRV IRUPDGDV SRU SULPRV 9HMD R DUWLJR XP SUHSULQW DLQGD QmR SXEOLFDGR GH *UHHQ H 7DR HP KWWSDU[LYRUJDEVPDWK17 H R DUWLJR H[SRVLWyULR GH %U\QD .UD VREUH R DVVXQWR HP KWWSPDWKQRUWKZHVWHUQHGXaNUDSDSHUVSULPHVSGI

EUREKA! N21, 2005

28

Sociedade Brasileira de Matemtica

CENTRO DE MASSA E APLICAES GEOMETRIA


Emanuel Carneiro & Frederico Giro UFC Nvel Avanado
1. INTRODUO

Chamaremos de sistema de massas um conjunto de n pontos P , P2 ,..., P no plano, n 1 sendo que ao ponto Pk = ( xk , yk ) est associada uma massa mk \, de modo que

m1 + m2 + ... + mn 0. Definiremos o centro de massa desse sistema como sendo o ponto ( x, y) tal que: m1 x1 + m2 x2 + ... + mn xn m y + m2 y2 + ... + mn yn ; y= 1 1 , M M Onde M = m1 + m2 + ... + mn a massa associada a ele. x=
Notao:

Quando ao ponto (x, y) estiver associada uma massa m escreveremos (x, y)[m].
Observaes:

(i) Podemos interpretar fisicamente o centro de massa de um sistema como sendo o ponto onde ele concentra toda sua massa. Em termos prticos, isso nos ajuda a simplificar, por exemplo, problemas de Dinmica onde h aplicaes de foras sobre o sistema. (ii) Podemos considerar os pontos em massa de um sistema anlogo.

\n . Neste caso, o clculo do centro de

(iii) Claramente o centro de massa nico.

2. PROPRIEDADES BSICAS Proposio 1.

Seja (x, y)[m] o centro de massa do sistema

S1 = {( x1 , y1 )[m1 ], ( x2 , y 2 )[m2 ],..., ( xk , y k )[m k ]}, e seja (a, b)[N] o centro de massa do sistema S 2 = {( a1 , b1 )[ n1 ], ( a2 , b2 )[n2 ],..., ( al , bl )[ nl ]}. Ento, se
EUREKA! N21, 2005

29

Sociedade Brasileira de Matemtica

M + N 0, o centro de massa do sistema S = S1 S 2 o centro de massa do sistema {( x , y )[ M ], (a , b )[ N ]}.


Demonstrao:

Por definio o centro de massa do sistema

S = S1 S 2 M x + Na M +N

o ponto

( X , Y )[ M + N ], onde:

X =

k i =1

mi xi + lj =1 n j a j M +N

que justamente a primeira coordenada do centro de massa do sistema {( x , y )[ M ], (a , b )[ N ]}. Para a segunda coordenada anlogo. A proposio acima nos d um algoritmo para calcular o centro de massa de um sistema com n pontos. Para isso tomamos dois pontos ( x1 , y1 )[m1 ] e

( x2 , y 2 )[ m2 ] quaisquer desse sistema e os substitumos pelo seu centro de massa com a massa m1 + m2 . Recamos assim num sistema com n 1 pontos e
continuamos o processo. Assim o clculo de centros de massa resume-se apenas ao caso n = 2, que estudamos a seguir:
Centro de massa de um sistema com duas massas

O centro de massa ( x , y )[ M ] de um sistema {( x1 , y1 )[m1 ], ( x 2 , y 2 )[m 2 ]} colinear com os pontos ( x1 , y1 ) e ( x2 , y 2 ) pois

x1 x2 x

y1 y2 y

1 1 = x1 y 2 + xy1 + x2 y xy 2 x2 y1 x1 y 1 m x + m2 x2 m1 y1 + m2 y2 = x1 y2 + 1 1 y1 + m1 + m2 m1 + m2 m x + m2 x2 m1 y1 + m2 y2 1 1 y 2 x2 y1 m1 + m2 m1 + m2 =0 x2 x1

E alm disso se chamamos ( x1 , y1 ) = A, ( x2 , y 2 ) = B e ( x , y ) = G vale que:


EUREKA! N21, 2005

30

Sociedade Brasileira de Matemtica

m1 AG + m2 BG = 0
tal fato deixado como exerccio para o leitor.
Observao:

JJJG

JJJG

Pela equao acima distinguimos alguns casos: As duas massas tm o mesmo sinal. Nesse caso o ponto G est entre A e B e vale que m1 AG = m2 BG . As duas massas tm sinais contrrios. Nesse caso G est fora do segmento AB e vale que m1 AG = m2 BG .
3. APLICAES GEOMETRIA Exemplo 1: Vamos tomar um tringulo ABC qualquer e pr massas iguais em seus

trs vrtices, ou seja consideraremos o sistema A[ p ], B[ p ], C [ p ]. Chamaremos de G o centro de massa desse sistema. Como encontrar o ponto G? (hummm) denotaremos C.M. = centro de massa. A[p]

P G

B[p]

M[2p]

C[p]

Vamos usar a proposio da seo anterior. O.C.M. de B[ p ] e C [ p ] o seu ponto mdio M. Podemos ento trocar B[ p ] e C [ p ] por M [2 p ]. Logo o ponto G ser o C.M. de A[ p ] e M [2 p ], que est sobre AM e divide AM na razo

AG 2 = . GM 1
EUREKA! N21, 2005

31

Sociedade Brasileira de Matemtica

Sejam N e P os pontos mdios de AC e AB. De modo anlogo poderamos ter provado que G BN e que G CP. Esta uma demonstrao diferente que as trs medianas concorrem em G, que portanto o baricentro do tringulo. Alm disso, segue do exposto acima que:

AG BG OG 2 = = = GM GN GP 1
Exemplo 2: Denote por a, b, c, os lados do tringulo ABC da maneira usual.

Vamos pr agora massas nos vrtices do tringulo proporcionais aos lados opostos, ou seja, considere o sistema A[ a ], B[b], C[ c ]. Seja I o C.M. desse sistema. Voc merece um prmio se descobrir quem I A[a]

K L I

B[b]
A[a]

J[b+c] a

C[c]

O raciocnio igual ao do exemplo anterior. O C.M. de B[b ] e C [c ] um ponto J

JB c = , ou seja, J o p da bissetriz interna. Logo I ser o JC b AI b + c = . C.M. de A[ a ] e J [b + c ]. Tiramos da que I AJ e que IJ a


no lado BC tal que Sejam BL e CK bissetrizes internas. De modo anlogo poderamos ter provado que I BL e que I CK , o que mostra que I o incentro. As razes saem de graa:

BI a + c CI a + b = = ; IL b IK c
Exemplo 3: Seja p o semipermetro do tringulo. Agora uma novidade: o sistema de massas ser A[ p a ], B[ p b], C[ p c]. Seja N o C.M. desse sistema. Voc realmente merece um prmio se descobrir quem o N.
EUREKA! N21, 2005

32

Sociedade Brasileira de Matemtica

A[p a]

pb Z

pc

Y N pa pa

B [p b]

pc

X[a]

pb

C[p c]

O C.M. de B[ p b ] e C [ p c ] um ponto X sobre o lado BC tal que

BX p c = , donde conclumos que BX = p c e que CX = p b. Este CX pb


ponto X onde o exincrculo relativo ao lado a toca este lado (como referncia sobre este fato podemos indicar [1]). Logo N ser o C.M. de A[ p a ] e

X [ p c + p b] = X [ a ].
Portanto N AX e

AN a = . Se considerarmos os pontos Y e Z onde os NX pa

exincrculos relativos aos lados b e c tocam estes lados, respectivamente, podemos mostrar que N BY e N CZ . Concluso: AX, BY e CZ so concorrentes em N que chamado Ponto de Nagel do ABC . Ora, ora, poderamos saber disso usando o teorema de Ceva (veja por exemplo [3]). Calma, o melhor ainda est por vir. As razes aqui so cortesias para ns:

BN b CN c = = ; NY p b NZ p c
O prximo resultado foi o que nos motivou a escrever este artigo. Ele mostra toda a beleza desta teoria, enquanto outros mtodos so ineficazes. Para uma demonstrao
EUREKA! N21, 2005

33

Sociedade Brasileira de Matemtica

completa (e bastante extensa) do prximo resultado usando a geometria plana clssica, veja [2].
Teorema 3.1. No ABC considere os pontos I, G e N como definidos acima.

Vale ento que I, G e N so colineares e ainda:

NG 2 = 1 GI
Prova: Seja p o semipermetro do tringulo. Considere um sistema de massas

A[ p ], B[ p ], C [ p ]. J sabemos que o C.M. desse sistema o baricentro G.


Fazendo uso da proposio 1, podemos dividir esse sistema em dois subsistemas S1 = A[ a ], B[b ], C [c ] e S 2 = A[ p a ], B[ p b ], C [ p c ]. O C.M. de S1 o incentro I com massa [a + b + c ] = [2 p ], enquanto o C.M. de S 2 o ponto da Nagel N com massa [ p a + p b + p c ] = [ p ]. Logo G ser o C.M. de

I [2 p ], N [ p ] o que implica I, N, G colineares (com G entre I e N) e ainda pela


equao do momento linear:

NG 2 = 1 GI
A[p] =[a + (p a)]

I[p] x
2y

O
2x

N[2p]

B[p] =[b + (p b)]

C[p] =[c + (p c)]

Corolrio 3.1.1. Em um tringulo qualquer ABC, sejam I,G,N como acima, O o circuncentro e I o ortocentro. Ento os pontos I, O, N, H formam um trapzio.

EUREKA! N21, 2005

34

Sociedade Brasileira de Matemtica

Prova: Sabemos que H, G, O so colineares (reta de Euler) e que:

HG 2 = GO 1
Segue ento do teorema anterior que IO paralelo a NH, logo I, O, N, H formam um trapzio, cujo encontro das diagonais G. Podemos aplicar estes mtodos do centro de massa em problemas que envolvem o ortocentro, o baricentro e os exincentros, para saber que massas devem estar nos vrtices, veja o problema 1. Divirta-se resolvendo estes problemas. Vale usar tudo, mas experimente a sua mais nova arma.
4. PROBLEMAS RELACIONADOS Problema 1

,B ,C (a) Verifique que o sistema A tem como C.M. o cos A cos B cos C ortocentro do tringulo.
(b) Verifique que o sistema A[ sen 2 A], B[ sen 2 B ], C [ sen 2C ] tem como C.M. o circuncentro. (c) Prove que o C.M. do sistema A[ a ], B[b], C [c ] o exincentro relativo ao lado a. Verifique os anlogos para os outros exincentros.
Problema 2

Sejam A, B, C, D pontos concclicos. Sejam G A , GB , GC , G D os baricentros dos tringulos BCD, ACD, ABD e ABC. Prove que G A , GB , GC , G D so concclicos.
Problema 3

Sejam ABCD um quadriltero no espao de forma que AB, BC, CD e DA sejam tangentes a uma esfera nos pontos X, Y, Z, W. Prove que estes pontos so coplanares.
Problema 4

Sejam X, Y e Z os pontos onde o incrculo do tringulo ABC toca os lados BC, AC e AB, respectivamente. Mostre que o incentro de ABC est sobre a reta que passa pelos pontos mdios de BC e AX. (veja uma soluo em [5]).
EUREKA! N21, 2005

35

Sociedade Brasileira de Matemtica

Problema 5

Considere 6 pontos em uma dada circunferncia. Tomamos trs destes pontos e marcamos seu baricentro G1 . Em seguida, marcamos o ortocentro H 2 dos outros trs pontos e traamos o segmento G1 H 2 . Mostre que todos os segmentos G1 H 2 passam por um ponto fixo.
Problema 6
6 = 20 3

possveis

l l em ABCD pelas bissetrizes de DPC e CQB , um losango.


Problema 7

Seja ABCD um quadriltero convexo inscritvel com os lados opostos AD e BC se encontrando em P, e AB e CD em Q. Prove que o quadriltero EFGH, determinado

Seja PABC um tetraedro e sejam A1 , B1 , C1 os pontos mdios das arestas BC, AC e AB, respectivamente. Seja um plano paralelo face ABC que intercepta as arestas PA, PB, PC nos pontos A2 , B2 , C 2 respectivamente. (a) Prove que A1 A2 , B1 B2 , C1C 2 concorrem em um ponto D. (b) Determine o lugar geomtrico dos pontos D quando varia.
Problema 8

(a) Considere 4 pontos que formam um sistema ortocntrico (cada um o ortocentro do tringulo formado pelos outros trs). Ponha massas iguais nesses 4 pontos. Prove que o centro de massa o centro do crculo dos nove pontos de cada um desses 4 tringulos (veja [1] e o problema proposto No. 107 na pgina 61). (b) (Beltrami) Prove que o C.M. do sistema formado pelo incentro e pelos trs exincentros com massas iguais o circuncentro.
Problema 9

l encontrando em P, e AB e CD em Q. Prove que as bissetrizes dos ngulos DPC e

Seja ABCD um quadriltero convexo inscritvel com os lados opostos AD e BC se

l CQB e a reta que une os pontos mdios das diagonais do quadriltero ABCD

(diagonal de Euler) concorrem.

EUREKA! N21, 2005

36

Sociedade Brasileira de Matemtica

Problema 10

(Banco IMO/97) No ABC acutngulo, sejam AD, BE alturas e AP, BQ bissetrizes internas. Sejam I e O o incentro e o circuncentro do tringulo ABC, respectivamente. Prove que os pontos D, E e I so colineares se e somente se P, Q e O so colineares.
Agradecimentos: A nosso amigo Carlos Shine pela primeira verso digitada deste

material, na Semana Olmpica de 2001, em Salvador - BA.

REFERNCIAS: [1] Coxeter, H.S.M.; Greitzer, S.L., Geometry Revisited, MAA, 1967. [2] Johnson, R.A., Advanced Euclidean Geometry, Dover Publications, 1960. [3] Castro, L.G.M., Introduo Geometria Projetiva, Eureka!, vol 8, pp 16-27, 2000. [4] Honsberger, R., Mathematical Morsels, MAA, 1978. [5] Moreira, C.G.T., Wagner, E., 10 Olimpadas Iberoamericanas de Matemtica, OEI, 1996.

+ 1 p SULPR" 4XH 28433 2 (VWH Q~PHUR GH  GtJLWRV p R TXLQWR PDLRU SULPR FRQKHFLGR H IRL GHVFREHUWR HP  ,VWR PRVWUD TXH  QmR p XP Q~PHUR GH 6LHUSLQVNL LVWR p XP QDWXUDO tPSDU N WDO TXH k 2 + 1 p FRPSRVWR SDUD WRGR n `  YHMD D (XUHND  SiJ   UHGX]LQGR SDUD  R Q~PHUR GH QDWXUDLV PHQRUHV TXH  TXH p R PHQRU Q~PHUR GH 6LHUSLQVNL FRQKHFLGR  VREUH RV TXDLV QmR VH VDEH VH VmR Q~PHURV GH 6LHUSLQVNL RX QmR FRQMHFWXUDVH TXH QHQKXP GHOHV VHMD           H  9HMD KWWSZZZVHYHQWHHQRUEXVWFRP SDUD PDLV LQIRUPDo}HV LQFOXVLYH VREUH FRPR DMXGDU D SURYDU HVVD FRQMHFWXUD 
7830457 n EUREKA! N21, 2005

9RFr VDELD

37

Sociedade Brasileira de Matemtica

SEQNCIA DE FIBONACCI
Ccero Thiago B. Magalhes Nvel Avanado
INTRODUO

O nome seqncia de Fibonacci, foi dado pelo matemtico francs Edouard Lucas no sculo XIX. Porm, a seqncia surgiu de um problema que estava proposto na obra "Liber Abaci" de Leonardo de Pisa (1180 1250), conhecido como Fibonacci. O problema era o seguinte: "Um homem pe um casal de coelhos dentro de um cercado. Quantos pares de coelhos sero produzidos em um ano, se a natureza desses coelhos tal que a cada ms um casal gera um novo casal, que se torna frtil a partir do segundo ms?" Depois de sculos de trabalho, possvel hoje citar uma quantidade enorme de propriedades da seqncia do nmero de coelhos existentes aps n meses. O objetivo deste trabalho apresentar algumas propriedades bsicas desta seqncia.
Definio

A seqncia de Fibonacci definida da seguinte maneira: f1 = f 2 = 1 e f n = f n 1 + f n 2 , n > 2. Por convenincia, algumas vezes usaremos f 0 = 0.
Propriedades bsicas

(I) Para todo n 1 : f1 + f 2 + ... + f n = f n + 2 1 Prova: n = 1: f1 = f 3 1 Vamos supor q 1 e f1 + f 2 + ... + f q = f q + 2 1

n = q + 1 : f1 + f 2 + ... + f q + f q +1 = f q + 2 1 + f q +1 = f q + 3 1
(II) Se m 1 e n > 1, ento f n + m = f n 1 f m + f n f m +1 Prova: Vamos fazer induo sobre m: m = 1: f n +1 = f n 1 f1 + f n f 2 = f n 1 + f n (verdadeira)

m = 2 : f n + 2 = f n 1 f 2 + f n f3 = f n 1 + 2 f n = ( f n 1 + f n ) + f n = f n + f n +1
(verdadeira) Seja q > 2 e suponhamos a propriedade vlida para todo k , 2 k < q , e para todo n > 1. Esta suposio, mais o fato de que a propriedade vale tambm para k = 1, nos garante:
EUREKA! N21, 2005

38

Sociedade Brasileira de Matemtica

f n + ( q 2) = f n 1 f q 2 + f n f q 1 f n + ( q 1) = f n 1 f q 1 + f n f q
Somando membro a membro essas igualdades e levando em considerao a frmula recursiva que define ( f n ) :

f n + q = f n 1 f q + f n f q +1
Ou seja, a frmula vale tambm para q, sempre que n > 1. O princpio da induo nos garante ento que vale para todo m 1 e qualquer n > 1. (III) Dois nmeros de Fibonacci consecutivos f n e f n +1 so primos entre si. A prova fica como exerccio. (IV) Se m | n, ento f m | f n . Prova: Por hiptese n = mq, para algum q `. Usaremos induo sobre r. Se q = 1, ento m = n e fcil ver que f m | f n . Seja q 1 e admitamos que

fm | f

mq

Ento, usando a propriedade (II):

f m ( q +1) = f mq + m = f mq 1 f m + f mq f m +1
Como f m | f mq 1 f m e f m | f mq f m +1 (pois, pela hiptese de induo, fm divide

f mq ), ento f m divide a soma desses dois produtos. Ou seja: f m | f m ( q +1) .


(V) Seja d = mdc(m, n), ento mdc ( f m , f n ) = f d . Prova: Induo em m + n. Se m = 1, mdc(m, n) = 1 e mdc(fm, fn) = mdc(1, fn) = 1 = f1. Se m + n = 2 trivial. Se m = n no h o que provar. Se 2 m < n, f n = f m + ( n m ) = f m 1 f n m + f m f n m + 1 m dc ( f m , f n ) =

mdc ( f m , f m 1 f n m ) = m dc ( f m , f n m ), que igual, por hiptese de induo a f mdc ( m , n m ) = f mdc ( m , n ) .


Veja tambm a soluo do problema proposto No. 92 na Eureka! 20, pp 55 57.
2 (VI) Seja x = x + 1, ento, para n = 2, 3, ns temos que

( III )

x n = f n x + f n 1 .
Prova: trivial o caso n = 2. E se x = f n x + f n 1 para algum n 2, ento
n

x n +1 = x n x = ( f n x + f n 1 ) x = f n x 2 + f n 1 x
EUREKA! N21, 2005

39

Sociedade Brasileira de Matemtica

= f n ( x + 1) + f n 1 x = ( f n + f n 1 ) x + f n = f n +1 x + f n ,
como desejado!
2 Vejamos que as razes de x = x + 1 so os nmeros =

1+ 5 1 5 . e= 2 2

Ento, para todo n = 2, 3, ns temos

n = f n + f n 1
e

n = f n + f n 1 .
Subtraindo as duas ltimas equaes temos que = f n ( ), e notando
n n

que =

5 , ns encontramos a frmula de Binet n n . fn =

Problema 1

Seja =
Soluo:

1+ 5 . Determine todos os n ` tais que n n 2 seja inteiro. 2


2

Note que raiz do polinmio p ( x ) = x x 1, com isso e usando a expresso da propriedade (V) temos que:

n n 2 = f n + f n 1 n 2 = ( f n n 2 ) + f n 1 .
Uma vez que irracional, segue da igualdade acima que n s ser
n 2

inteiro quando f n n = 0, e nosso problema equivale a determinar todos os


2 2 n ` tais que f n = n . Para tanto, observe que: f1 = 1, f 2 = 1, f 3 = 2, f 4 = 3, f 5 = 5, f 6 = 8, f 7 = 13, f 8 = 21, f 9 = 34, f10 = 55, f11 = 89, f12 = 144, f13 = 233, f14 = 377.

Assim,

f12 = 12 2 e f13 > 13 2 , f14 > 14 2. Por

outro lado, se

f n > n2 e

f n +1 > (n + 1) 2 , ento f n + 2 = f n +1 + f n > n 2 + (n + 1) 2 > (n + 2) 2 , desde que n > 3,


EUREKA! N21, 2005

40

Sociedade Brasileira de Matemtica


2 donde segue por induo que f n > n para n > 13. Assim, as nicas solues so n = 1 e n = 12.

Problema 2:

Sejam n e k dois inteiros positivos quaisquer. Ento entre duas potncias k k +1 consecutivas n e n no podemos ter mais que n nmeros de Finonacci. Sugesto: use a propriedade (I)
Problema 3:

Seja f n a seqncia de Fibonacci ( f1 = 1, f 2 = 1, f n +1 = f n + f n 1 ). Calcule a srie

Problema 4:

n=2

fn . f n 1 f n +1

2 Ache a, se a e b so nmeros inteiros tais que x x 1 um fator de

ax17 + bx16 + 1.
Fortemente ligada seqncia de Fibonacci, e to interessante quanto, a seqncia de Lucas que definida da seguinte maneira:

Ln = f n 1 + f n +1 L1 = 1, L2 = 3, L3 = 4,...
Obs: fcil perceber que de acordo com a definio da seqncia de Lucas temos que L0 = 2. Usando a frmula de Binet temos que:

Ln = f n 1 + f n +1 =
Como =

n 1 n 1 n +1 n +1 = +

1 n1 n1 + + .
1 + = 5. Portanto,

1 1+ 5 1 5 + = 5 e , ento e = 2 2

Ln = n + n .
Problema 5:

Prove que f 2 n = f n Ln .

EUREKA! N21, 2005

41

Sociedade Brasileira de Matemtica

Problema 6:

Sejam L0 = 2, L1 = 1, e Ln + 2 = Ln +1 + Ln , para n 0, a seqncia de Lucas. Prove que, para todo m 1,

L
k =1

2k

= f 2m +1 ,

onde f n a seqncia de Fibonacci.


Problema 7:
3 Achar o termo geral pn se p0 = 1 e pn +1 = 5 pn 3 pn , para n 0.

Problema 8:

Todo natural pode ser unicamente escrito como soma de nmeros de Fibonacci distintos, de ndices no consecutivos e maiores que 1. (Teorema de Zeckendorff).
Problema 9:
93 19 93 Sejam a e b inteiros positivos tais que b divisvel por a e a divisvel por f f b19 . Prove que ( a 4 + b 8 ) n+1 divisvel por ( ab ) n para todo n > 1.

Problema 10:

Prove que nenhum nmero de Fibonacci potncia de 7.


Problema 11:

Sejam ( f n )

a seqncia de Fibonacci e, para todo inteiro positivo n,

Vn =
rea 1 .
2

f n2 + f n2+ 2 .

Prove que, para todo inteiro positivo n, Vn , Vn +1 , Vn + 2 so lados de um tringulo de

REFERNCIAS: [1] Ross Honsberger, Mathematical Gems III, MAA, 1985. [2] Hygino H. Domingues, Fundamentos da Aritmtica, Atual, 1991.

EUREKA! N21, 2005

42

Sociedade Brasileira de Matemtica

COMO QUE FAZ?


PROBLEMA 6 PROPOSTO POR RAFAEL ALVES DA SILVA (TERESINA - PI) da Olimpada Ibero-americana de Matemtica de 1999.

Seja n um inteiro maior que 10 tal que cada um dos seus dgitos pertence ao conjunto S = {1, 3, 7, 9}. Prove que n tem algum divisor primo maior ou igual a 11. Soluo: Como o ltimo dgito de n mpar, n mpar. Como o ltimo dgito de n no 0 nem 5, n no mltiplo de 5. Assim, se n no tivesse nenhum fator primo maior ou igual a 11, os nicos possveis fatores primos de n seriam 3 e 7. Mostraremos por induo que todo nmero natural cujos nicos fatores primos so 3 e 7 da forma n = 20k + r, onde k natural e r {1, 3, 7, 9} , e portanto o dgito das dezenas de n deve ser par, e logo no pode pertencer a S = {1, 3, 7, 9} (e assim, se todos os dgitos de n pertencem a S, devemos ter n < 10). Para isso, note que 1 = 20 0 + 1, e ainda n = 20k + 1 3n = 20 (3k) + 3 e 7n = 20 (7k) + 7 n = 20k + 3 3n = 20 (3k) + 9 e 7n = 20 (7k + 1) + 1 n = 20k + 7 3n = 20 (3k + 1) + 1 e 7n = 20 (7k + 2) + 9 n = 20k + 9 3n = 20 (3k + 1) + 7 e 7n = 20 (7k + 3) + 3, e nossa afirmao est provada.
PROBLEMA 7 PROPOSTO POR RAFAEL ALVES DA SILVA (TERESINA - PI) da Olimpada da Bielorrsia de 2000.

Seja M = {1, 2, 3,..., 40}. Ache o menor inteiro n para o qual possvel particionar M em M subconjuntos disjuntos tais que, sempre que a, b e c (no necessariamente distintos) pertencem ao mesmo subconjunto, ento a b + c. Soluo: Mostraremos que n = 4. Vamos ver como construir um exemplo de uma partio de n em 4 conjuntos de acordo com o enunciado, a partir de exemplos menores: temos a partio {1} = {1}, com n = 1, para M = {1}; a partio {1, 2, 3, 4} = {1, 4} {2, 3}, com n = 2, para M = {1, 2, 3, 4}; a partio {1, 2, 3, 4, 5, 6, 7, 8, 9, 10, 11, 12, 13} = {1, 4, 10, 13} {2, 3, 11, 12} {5, 6, 7, 8, 9} com n = 3, para M = {1, 2,,13}; a idia , dada uma partio {1, 2,, k} =
EUREKA! N21, 2005

43

Sociedade Brasileira de Matemtica

A A2 ... An , obter uma partio {1, 2,..., 3k + 1} = B1 B2 ... Bk Bk +1 , 1 tomando Bk+1 = {k +1, k + 2,...,2k +1} e, para i k, Bi = Ai (Ai + (2k +1)). Assim, obtemos, para M = {1,2,...,40}, M = {1,4,10,13,28,31,37,40}{2,3,11,12,29,30,38,39} {5,6,7,8,9,32,33,34,35,36}{14,15,16,17,18,19,20,21,22,23,24,25,26,27}, e fcil
verificar que esta partio satisfaz a condio do enunciado. Para mostrar que no possvel obter uma tal partio de M em apenas 3 subconjuntos, suponha que M = A B C. Como M tem 40 elementos, algum dos conjuntos A, B ou C tem pelo menos 14 elementos. Digamos que A contenha os 14 elementos x1 < x2 < ... < x14 . Os 13 elementos x14 x1 , x14 x2 ,..., x14 x13 devem pertencer a B C, pois, se x14 xj = y A, ento x14 = xj + y, com y , x j , x14 A, contradio. Assim, B ou C contm pelo menos 7 desses 13 elementos, digamos y1 < y2 < ... < y7 , onde yi = x14 xji , onde j7 < j6 < ... < j1. Suponhamos que esses elementos pertencem a B. Temos ento e, ( x14 x j3 ) ( x14 x ji ) como

y3 y2 C e y3 y1 C, pois, se y3 y j = z B, para algum j {1, 2}, y3 = y j + z , com y3 , y j , z B , absurdo,

y3 yi = ( x14 x j3 ) ( x14 x ji ) = x ji x j3 , se

y3 yi = z A, para algum i {1, 2}, teramos x ji = x j3 + z , com xji , xj3 , z A ,


absurdo. Vamos agora considerar o elemento y2 y1 > 0. Como y1 e y2 pertencem a B, y2 y1 no pode pertencem a B. Como y2 y1 = ( y3 y1 ) ( y3 y2 ), com

( y3 y2 ), ( y3 y1 ) C, y 2 y1 no pode pertencer a C. Finalmente, como y2 y1 = (x14 x j2 ) (x14 x j1 ) = x j1 x j2 , com x j1 A e xj2 A, y2 y1 no pode


pertencer a A, absurdo, pois y2 y1 M = A B C.
Publicamos a seguir as solues dos problemas 121 e 195 da seo "Olimpadas ao redor do mundo", por sugesto de Bruno, da Espanha. 121. (Rssia-2001) Os valores da funo quadrtica f (x ) = x 2 + ax + b para dois

inteiros consecutivos so os quadrados de dois inteiros tambm consecutivos. Mostre que os valores da funo quadrtica so quadrados perfeitos para todos os inteiros coincide com o conjunto dos valores de g para os inteiros. Soluo: Suponha f (m) = k 2 e f (m + 1) = (k + 1) 2 , com m e k inteiros. Seja g ( x) = f ( x + m). O conjunto dos valores de f para os inteiros.
EUREKA! N21, 2005

44

Sociedade Brasileira de Matemtica

Temos g ( x) = x 2 + cx + d para certos valores de c e d. Temos d = g (0) = f ( m) = k 2 e 1 + c + d = g (1) = f (m + 1) = ( k + 1) 2 , donde d = k 2 e c = (k + 1) 2 1 k 2 = 2k , ou seja, g ( x) = x 2 + 2kx + k 2 = ( x + k ) 2 , e logo g(x) um quadrado perfeito para todo x inteiro.
195.(Eslovnia-2002) Sejam M o ponto mdio da base AB do trapzio ABCD; E um

ponto interior ao segmento AC tal que BC e ME intersectam-se em F; G o ponto de interseo de FD e AB; H o ponto de interseo de DE e AB. Mostre que M o ponto mdio do segmento GH. Soluo: Podemos supor sem perda de generalidade, aplicando uma transformao afim, que A = (0, 0), B = (1, 0), C = (1, 1), e logo D = (a, 1), para algum a < 1. Ento 1 M = ,0 e E = (t, t), para algum t (0,1). Como F pertence a BC, F = (1, y) para 2 1 algum y, e, supondo t , como F pertence a ME, y 1 = t t 1 , donde 2 2 2 1 t (caso t = , BC e ME so paralelas e no se intersectam, logo no y= 2t 1 2 existiria o ponto F). Como H pertence a AB, H = (x, 0) para um certo x. Como H 1 t t (1 a ) , donde (1 t ) x = t (1 a ), ou seja, x = . pertence a DE, = 1 t tx ax Finalmente, como G pertence a AB, G = (z, 0) para um certo z, e como G pertence a 1 (2t 1) at t (2t 1) t 1 at (2t 1) , donde , ou seja, z = . = z = FD, 1 z 2t 1 2t 1 1 t az x + z 1 1 t 1 G+H Assim, , ou seja, M o ponto mdio do = = , e logo M = 2 2 1 t 2 2 segmento GH.

( ) (

EUREKA! N21, 2005

45

Sociedade Brasileira de Matemtica

SOLUES DE PROBLEMAS PROPOSTOS


Publicamos aqui algumas das respostas enviadas por nossos leitores.

90) Prove que, para todo inteiro positivo n e para todo inteiro no nulo a, o polinmio xn + axn1 + axn2 + ... + ax 1 irredutvel, i.e., no pode ser escrito como o produto de dois polinmios no constantes com coeficientes inteiros.
SOLUO (baseada em um argumento de Artur Avila):

1 P (x) = xn + axn1 + axn2 + ... + ax 1. Como P a (x) = xn P , Pa ( x) a a x irredutvel se e somente se P a ( x) irredutvel. Assim, podemos supor sem perda de generalidade que a um inteiro positivo. Nesse caso, P (0) = 1 e a P (1) = (n 1)a > 0, n 2. a claro que para n = 1 o polinmio P ( x) = x 1 irredutvel. a
Seja Para n 2 , deve existir ( 0,1) com P ( ) = 0. a

Qa (x) = Pa (x) (x 1) = (x 1)(xn 1) + ax(xn1 1) = xn+1 + (a 1)xn (a +1)x +1. Como raiz de P ( ) = 0. a
Seja Seja

Qa (x) = Pa (x) (x 1) = (x 1)(xn 1) + ax(xn1 1) = xn+1 + (a 1)xn (a +1)x +1. Como raiz de P ( x), tambm raiz de Qa ( x). Seja ento a Q (x) 1 R(x) = a = xn + (a + 1)xn1 +(a + 1)xn2 +2 (a + 1)xn3 +... +n2 (a + 1)x . x Temos R(1) = Q (1) (1 ) = 0, e, se raiz de P ( x), raiz de Qa ( x) e de a R(x). Mostraremos que > 1 para todo ^ , tal que raiz de Pa (x). + n 2 (a + 1) =
Como raiz de R(x), temos n + ( a + 1) n 1 + ( a + 1) n 2 + ... +

1 . Se 1, n + ( a + 1) n 1 + ( a + 1) n 2 + ... + n 2 ( a + 1) + ( a + 1)
n n 1

+ ... + n 2 ( a + 1) 1n + ( a + 1)1n 1 + ... +

EUREKA! N21, 2005

46

Sociedade Brasileira de Matemtica

1 1 1 = = , valendo a igualdade se, e somente se, = = 1, mas Pa (1) 0, donde 1, absurdo (na verdade, mostramos que todas as raizes de R ( x ) tm mdulo maior ou igual a 1; em particular, raiz simples de Pa ( x ), caso contrrio (0,1) seria raiz de R ( x)). Suponha agora que Pa ( x ) = f ( x ) g ( x), onde f e g so polinmios no constantes com coeficientes inteiros. Como o coeficiente constante de Pa ( x ) 1, os + n 2 (a + 1) 1 = R(1) +
coeficientes constantes de f(x) e de g(x) pertencem a {1, 1} (e podemos supor sem perda de generalidade que seus coeficientes lderes so iguais a 1). Assim, o produto das razes de g(x) (e de f(x)) pertencem a {1, 1}. Por outro lado, podemos supor sem perda de generalidade que raiz de f ( x ). Como todas as razes de g(x) so razes de Pa ( x ) distintas de , elas tm todas mdulo maior que 1, e logo seu produto no pode pertencer a {1, 1}, absurdo. 95) "Resta-Um" um jogo de tabuleiro na qual as peas ocupam um tabuleiro formando parte de um reticulado retangular (na verdade, existem variaes em tabuleiros de reticulado triangular). O nico movimento permitido consiste em tomar duas peas em casas adjacentes vizinhas a uma casa vazia, e fazer a pea mais distante da casa vazia pular sobre a outra pea, ocupando a casa vazia. A pea pulada retirada.

(esse movimento pode ser feito para a direita, para a esquerda, para cima ou para baixo). Agora imagine um tabuleiro que um reticulado retangular infinito e uma reta que contm uma linha do reticulado, dividindo-o em dois lados. Todas as casas de um dos lados da linha esto vazias e cada casa do outro lado da linha pode ou no ter uma pea. Quantas peas, no mnimo, precisamos para chegar a uma casa do lado vazio do tabuleiro, a uma distncia n da linha ? Abaixo indicamos uma casa a distncia n, para n = 1,2,3,4,5.

EUREKA! N21, 2005

47

Sociedade Brasileira de Matemtica

5 4 3 2 1

SOLUO DE BERNARDO FREITAS PAULO DA COSTA (RIO DE JANEIRO RJ)

Vamos botar eixos no quadriculado, de acordo com a posio: Y

"Fronteira" X

Assim, a casa marcada com 9 (0, 0), sua vizinha esquerda (1, 0) e acima (0, 1). Supomos, sem perda de generalidade, que a pea final termina na casa (0, n) para n = 1, 2, 3, 4, 5. Usando isto, vemos que o movimento deveria "tender para o centro e para cima". Com esta idia, vamos associar energias s peas nos quadrados do tabuleiro, atravs da seguinte funo:

2 = + 1, > 1 E (a, b) = onde 2 + = 1, 0 < < 1


b a EUREKA! N21, 2005

48

Sociedade Brasileira de Matemtica

Como as equaes que os definem so simtricas, temos = 1

S para conferir:

1 5 2

1+ 5 >1 1+ 5 2 = 2 1 5 <0 2 5 1 (0,1) 5 1 2 = 2 1 5 <0 2

1 5 2

Assim, se fizermos um movimento

a 2

a 1

a 2

,a0

ou um movimento

,a0 a +2 a+ na direo do centro, estaremos trocando uma energia b + b 1 por

a +1

a +2

a +2

b a , que so iguais por construo.


J se fizermos

a +1

a +2

a +2

,a>0 ou o contrrio (com a < 0) trocaremos energia com perdas, pois a a + 2 (para a > 0; se a < 0 a a 2 ). Para movimentos verticais, vemos que

EUREKA! N21, 2005

49

Sociedade Brasileira de Matemtica

b+2

b+1

conserva energia b + b +1 = b + 2
a a
b+2

) e que

b+1

perde (pois b + 2 > b e b+1 > 0 ). Assim, a energia das peas no tabuleiro no aumenta. Vejamos a energia mxima do tabuleiro: ET = E (coluna central) + E (quadrantes negativos) = E(coluna central) + 2E(1 quadrante negativo) =

= =

b = 0

b 0 + 2 E (coluna em a)
a =1

0 a + 2 b b = a =1 b = + a = b + 2 b b =0 a =1 b =0 1 1 1 1 1 1 a Mas 2 = + 1 1 = + 2 1 = 2 = + 2 1 1 a =1 1 1

1 1 + 2 a 1 2 1 2 a =1

2 = 2 1 + 2 a = 2 1 + 1 a =1

EUREKA! N21, 2005

50

Sociedade Brasileira de Matemtica

Como 2 + = 1, temos 1 = 2 , e assim

2 ET = 2 1 + 2 = 2 1 + 2 = 2 (1 + 2 ) = 2 (1 + + ) 2 2 = ( + ) = 2 3 = 5
Ora, como a energia total do tabuleiro 5 , se tivermos um nmero finito de movimentos (para poder chegar em (0, 5) de verdade) teremos usado um conjunto finito de peas e portanto de energia estritamente menor do que 5 . Desta forma, impossvel chegar a esta casa com movimentos "resta-um". Decomponhamos n para n = 1, 2, 3, 4 para sabermos o nmero de peas

a + 2 = a +1 + a necessrias. Vale lembrar as relaes: a = a +1 + a +2 1 = 1 + = 1 + 2 ( = +1) = 1 1 = 1 + 2 peas:


"Fronteira" OK

Altura 2: 2 = + 1 = 1 + (1 + )

s temos 1 pea com energia 1 s temos 3 peas com energia

= 1 + ( + 2 + )

4 peas:

s temos 5 peas com energia 2 (em geral, para h 0, s temos 2h + 1 peas com energia h )
"Fronteira"

EUREKA! N21, 2005

51

Sociedade Brasileira de Matemtica

"Fronteira"

"Fronteira"

Altura 3: 3 = 2 + = 2 + 1 = 3 + 2 = 1 + 2 + (2)

= 1 + 2 + (2 + 2 2 ) = 1 + 3 + 2 2 + () = 1 + 3 + 3 2 + 3 8 peas:
vai (0, 2)

"Fronteira"

"Fronteira"

OK

"Fronteira"

EUREKA! N21, 2005

52

Sociedade Brasileira de Matemtica

Vamos ao ltimo caso (altura 4):

4 = 3 + 2 = 2 2 + = 3 + 2 = 5 + 3
= 1 + 3 + (4) = 1 + 3 + (4 + 4 2 ) = 1 + 3 + 4 2 + (4 2 + 43 ) = 1 + 3 + 5 2 + 4 3 + (3 2 ) = 1 + 3 + 5 2 + 7 3 + 3 4
Teoricamente, isto daria 1 + 3 + 5 +7 + 3 = 19 peas para chegar a altura 4. Entretanto, isto impossvel. Vejamos: Pintemos o tabuleiro com 3 cores em diagonal
B C A B C A B C A B C C A B C A B C B C A B C A B C A B C A B C A B C

A B C A B C A

A B C A B C

A "Fronteira" B A C B A C B A

Toda vez que realizarmos um movimento, tiramos uma pea de duas cores adjacentes e a terceira cor recebe uma pea a mais. Assim, invertemos a paridade dos 3 simultaneamente. Portanto, a paridade relativa se mantm. As casas dentro da regio indicada estaro todas ocupadas (suas energias somam 1 + 3 + 5 2 + 7 3 ). A soma das cores 5A + 7B + 4C Se a soma final 1A + 0B + 0C, temos que a paridade de B e C a mesma, e oposta paridade de A, logo das 3 4 que restam, temos uma quantidade par da cor A, uma quantidade mpar de cor B e uma quantidade para da cor C. Assim, temos as seguintes possibilidades: 2A + 1B + 0C 0A + 3B + 0C 0A + 1B + 2C Mas s h 1 casa B valendo 4 logo a possibilidade OA + 3B + 0C est excluda. Assim, conclumos que h 1 pea numa casa B, a nica do nvel 4 . Por simetria (ou seja, pintando o tabuleiro na outra direo : /// ),
EUREKA! N21, 2005

53

Sociedade Brasileira de Matemtica

temos que deve haver tambm uma pea na casa simtrica a essa casa B do nvel 4 em relao coluna central, a qual uma casa C, e logo a soluo OA + 1B + 2C. Assim, temos a seguinte configurao:

"Fronteira"

X ? X ? X ?

As casas marcadas com X so A ou simtricas a casas A, e logo esto proibidas. As casas com esto forosamente ocupadas, e das casas com ?, uma, e apenas uma, ser ocupada (para fazer 19). Isolemos a coluna central; sobra (pelo menos) um bloco consistindo apenas das seguintes peas: (ou do outro lado, por simetria igual).

Ora, a pea : , para ser aproveitada, deve tomar uma pea que est a sua direita e ir para a coluna 1. Entretanto, para terminarmos com peas apenas na coluna zero, as duas indicadas pelas setas devem tomar as peas que esto direita (pois seno no conseguiramos mandar todas as peas desse bloco para a coluna central usando apenas essas peas; se usssemos outras peas para isso perderamos energia). Nesse caso, as peas que sobram do lado esquerdo ficam todas na altura 0, e a pea que estava originalmente na coluna 0 e na coluna 1 no conseguira chegar na coluna central sem que haja perda de energia. Assim, no conseguimos (agora por impedimento de movimento, no de energia) chegar em (0, 4) com 19 peas. Mas possvel com 20 (fao 3 = 4 + 5 ):

EUREKA! N21, 2005

54

Sociedade Brasileira de Matemtica

"Fronteira"

"Fronteira"

"Fronteira"

"Fronteira"

"Fronteira" "Fronteira"

"Fronteira"

OK!!

96) No quadriltero ABCD os ngulos A, C e D medem 100 e o ngulo ACB mede 40. Demonstre que BC DA = ( BC + AB DA)2 .

EUREKA! N21, 2005

55

Sociedade Brasileira de Matemtica

SOLUO DE HEYTOR BRUNO E MARLON JNIOR (FORTALEZA CE)

A figura ilustra o problema:


A 20 80

60 D 100 60 C

40

AC sen100 = DA sen60 BC sen80 ACB (Lei dos Senos): = AC sen 60 AC BC = AC 2 = BC DA DA AC ACD (Lei dos Senos):
Assim,

BC DA = ( BC + AB DA)2 AC 2 = ( BC + AB DA)2 . Basta provar ento


que AC = BC + AB DA.

ACD (Lei dos Senos): DA = ACB (Lei dos Senos): BC =


Da, AC + DA = BC + AB

AC sen60 sen100 AC sen80 AC sen 40 e AB = sen60 sen 60

AC +

AC sen60 AC sen80 AC sen 40 = + sen60 sen60 sen100

EUREKA! N21, 2005

56

Sociedade Brasileira de Matemtica

sen100 + sen60 sen80 + sen 40 = . sen60 sen100

Usando o fato de que

p+q pq sen p + sen q = 2 sen cos , 2 2


A equao acima verdadeira se e somente se

2 sen80 cos 20 2 sen60 cos 20 = , o que obviamente verdade. sen100 sen 60


99) Num tringulo, a razo entre os raios das circunferncias circunscrita e inscrita

5 . Os lados do tringulo esto em progresso aritmtica e sua rea 2


numericamente igual ao seu permetro. Determine os lados do tringulo.
SOLUO DE KELLEM CORRA SANTOS (RIO DE JANEIRO RJ)
A

R 5 = 2 R = 5r r 2
R a

a+q

r B

aq

Perimetro de ABC 2 p = a + (a + q) + (a q) = 3a p = pr = A( ABC ) =

3a 2

a( a + q)(a q) 4R

EUREKA! N21, 2005

57

Sociedade Brasileira de Matemtica

3a a (a 2 q 2 ) a2 q2 a2 q2 3r = = 15r 2 = a 2 q 2 r= 2 4R 2R 5r
Como a rea numericamente igual ao permetro, temos:

(1)

pr = 2 p r = 2
Por outro lado,

A( ABC ) = =

p( p a )( p a q)( p a + q ) =

3a a a a q + q = 2 2 2 2

a2 1 a 3 q2 2 4

De (1), temos:

15r 2 = a 2 q 2 15 2 2 = a 2 q 2 60 = a 2 q 2 q 2 = a 2 60
Logo A( ABC ) =

a 3a 2 a a2 3 a 2 + 60 = 3 60 2 4 4 2 3a 2 = 3a 2

Tambm podemos escrever:

A( ABC ) = pr =
Igualando:

3a =

3a 2 a 3 60 2 4 3a 2 36 3a 2 3a 2 = 60 12 = 60 62 = 3 60 4 3 4 4 3a 2 a 2 = 16 4 a = 8, pois a > 0. 4

48 =
De (1):

15r 2 = a 2 q 2 60 = 64 q 2 q = 2
Logo, os lados do tringulo ABC so 6, 8 e 10. 101) a) Sejam ai , bi , ci reais positivos, para 1 i 3.

3 3 3 3 3 3 3 3 Prove que (a1 + a2 + a3 )(b13 + b2 + b3 )(c1 + c2 + c3 ) (a1b1c1 + a2b2c2 + a3b3c3 )3 .

b) Sejam a, b, c, x, y, z reais positivos. Prove que

EUREKA! N21, 2005

58

Sociedade Brasileira de Matemtica

a 3 b 3 c 3 ( a + b + c )3 + + . x 2 y 2 z 2 ( x + y + z)2
SOLUO DE MARCELO RIBEIRO DE SOUZA e WALLACE MARTINS (RIO DE JANEIRO RJ)

a) Como esta uma desigualdade homognea, podemos supor


3 3 3 (a1 + a2 + a3 ) = 1 3 3 (b13 + b2 + b3 ) = 1 3 3 (c13 + c2 + c3 ) = 1

Com isto nossa desigualdade passa a ser: 1 (a1b1c1 + a2b2 c2 + a3b3c3 )3 . Isto imediato de MA MG:

(a13 + b13 + c13 ) a1b1c1 3 3 3 3 (a2 + b2 + c2 ) a2b2c2 3 3 3 3 (a3 + b3 + c3 ) a3b3c3 3


Somando tudo e elevando ao cubo ficamos com:

1 ( a1b1c1 + a2b2 c2 + a3b3c3 ) cqd.


3

b) Este item decorre diretamente do anterior, tendo em vista que


3 3 3 1 3 1 3 1 3 1 3 1 3 1 3 a b c 3 3 3 3 3 3 2 + 2 + 2 x + y + z x + y + z x 3 y 3 z 3

1 1 1 1 1 1 a x3 x3 + b y3 y3 + c z3 z 3 2 2 2 x3 3 y z3
Portanto segue que

a 3 b3 c 3 ( a + b + c ) . 2 + 2 + 2 y z ( x + y + z )2 x
3

EUREKA! N21, 2005

59

Sociedade Brasileira de Matemtica

PROBLEMAS PROPOSTOS
Convidamos o leitor a enviar solues dos problemas propostos e sugestes de novos problemas para os prximos nmeros.

102) Voc recebe x metros de arame para cercar um terreno na forma de um tringulo pitagrico (os lados so nmeros inteiros), com a condio de que a medida do cateto menor seja 24 metros. Qual dever ser a medida do cateto maior e o comprimento do arame, a fim de que a rea seja: a) mxima? b) mnima? 103) Sejam A e B matrizes 2 2 com elementos inteiros. Sabendo que A, A + B, A + 2B, A + 3B e A + 4B so invertveis e que os elementos das respectivas inversas tambm so todos inteiros, mostre que A + 5B tambm invertvel e que os elementos da sua inversa tambm so inteiros. 104) ABC um tringulo. Mostre que existe um nico ponto P de modo que:
2 2 2

( PA ) + ( PB ) + ( AB )

= ( PB ) + ( PC ) + ( BC ) = ( PC ) + ( PA ) + (CA)
2 2 2 2 2

105) O baricentro do tringulo ABC G. Denotamos por g a , gb , gc as distncias desde G aos lados a, b e c respectivamente. Seja r o rdio da circunferncia inscrita. Prove que:

2r 2r 2r , gb , g c 3 3 3 g + gb + gc b) a 3 r
a) g a 106) Os polinmios P0 ( x, y, z ), P ( x, y, z ), P2 ( x, y, z ),... so definidos por 1

P0 ( x, y, z ) = 1 e Pm+1 ( x, y , z ) = ( x + z )( y + z ) Pm ( x, y , z + 1) z 2 Pm ( x, y, z ), m 0. Mostre que os polinmios Pm ( x, y , z ), m ` so simtricos em x, y, z, i.e., P ( x, y, z) = P ( x, z, y) = P ( y, x, z) = P ( y, z, x) = P ( z, x, y) = P ( z, y, x), m m m m m m


para quaisquer x, y, z.

EUREKA! N21, 2005

60

Sociedade Brasileira de Matemtica

107) a) Dado um tringulo qualquer, prove que existe um crculo que passa pelos pontos mdios dos seus lados, pelos ps das suas alturas e pelos pontos mdios dos segmentos que unem o ortocentro aos vrtices do tringulo (o chamado "crculo dos nove pontos"). b) Prove que, se X o centro do crculo dos nove pontos de um tringulo, H o seu ortocentro, O seu circuncentro e G seu baricentro, ento

JJJG 3 JJJG 1 JJJG OX = OG = OH . 2 2

Problema 102 proposto por Sebastio Vieira do Nascimento (Campina Grande PB); Problema 103 proposto por Carlos A. Gomes (Natal RN); Problemas 104 e 106 propostos por Wlson Carlos da Silva Ramos (Belm PA); Problema 105 selecionado da XXXV Olimpada Matemtica Espanhola, fase nacional, 1999 e enviado por Bruno Salgueiro Fanego (Espanha). Agradecemos tambm o envio das solues e a colaborao de: Carlos Alberto da Silva Victor Daniel Lopes Alves de Medeiros Gabriel Ponce Georges Cobiniano Sousa de Melo Glauber Moreno Barbosa Jnatas de Souza Jnior Rafael Silva Raphael Constant da Costa Nilpolis RJ Fortaleza CE Por e-mail Joo Pessoa PB Rio de Janeiro RJ Recife PE Teresina PI Rio de Janeiro RJ

Seguimos aguardando o envio de solues dos problemas propostos Nos. 89, 97, 98 e 100.

EUREKA! N21, 2005

61

Sociedade Brasileira de Matemtica

COORDENADORES REGIONAIS
Alberto Hassen Raad Amrico Lpez Glvez Amarsio da Silva Arajo Ana Paula Bernardi da Silva Antonio Carlos Nogueira Ali Tahzibi Benedito Tadeu Vasconcelos Freire Carlos Frederico Borges Palmeira Claus Haetinger Cleonor Crescncio das Neves Cludio de Lima Vidal Edson Roberto Abe lio Mega der Luiz Pereira de Andrade Eudes Antonio da Costa Florncio Ferreira Guimares Filho Ivanilde Fernandes Saad Jacqueline Fabiola Rojas Arancibia Janice T. Reichert Joo Bencio de Melo Neto Joo Francisco Melo Libonati Jos Carlos dos Santos Rodrigues Jos Cloves Saraiva Jos Gaspar Ruas Filho Jos Luiz Rosas Pinho Jos Vieira Alves Krerley Oliveira Licio Hernandes Bezerra Luzinalva Miranda de Amorim Mrio Rocha Retamoso Marcelo Rufino de Oliveira Marcelo Mendes Marilane de Fraga Sant'Ana Pablo Rodrigo Ganassim Ramn Mendoza Ral Cintra de Negreiros Ribeiro Ronaldo Alves Garcia Reginaldo de Lima Pereira Reinaldo Gen Ichiro Arakaki Ricardo Amorim Srgio Cludio Ramos Seme Guevara Neto Tadeu Ferreira Gomes Toms Menndez Rodrigues Valdenberg Arajo da Silva Valdeni Soliani Franco Vnia Cristina Silva Rodrigues Wagner Pereira Lopes (UFJF) (USP) (UFV) (Universidade Catlica de Braslia) (UFU) (USP) (UFRN) (PUC-Rio) (UNIVATES) (UTAM) (UNESP) (Colgio Objetivo de Campinas) (Colgio Etapa) (UNESPAR/FECILCAM) (Univ. do Tocantins) (UFES) (UC. Dom Bosco) (UFPB) (UNOCHAPEC) (UFPI) (Grupo Educacional Ideal) (Unespar) (UFMA) (ICMC-USP) (UFSC) (UFPB) (UFAL) (UFSC) (UFBA) (UFRG) (Grupo Educacional Ideal) (Colgio Farias Brito, Pr-vestibular) FACOS (Liceu Terras do Engenho) (UFPE) (Colgio Anglo) (UFGO) (Escola Tcnica Federal de Roraima) (UNIVAP) (Centro Educacional Logos) (IM-UFRGS) (UFMG) (UEBA) (U. Federal de Rondnia) (U. Federal de Sergipe) (U. Estadual de Maring) (U. Metodista de SP) (CEFET GO) Juiz de Fora MG Ribeiro Preto SP Viosa MG Braslia DF Uberlndia MG So Carlos SP Natal RN Rio de Janeiro RJ Lajeado RS Manaus AM S.J. do Rio Preto SP Campinas SP So Paulo SP Campo Mouro PR Arraias TO Vitria ES Campo Grande MS Joo Pessoa PB Chapec SC Teresina PI Belm PA Campo Mouro PR So Luis MA So Carlos SP Florianpolis SC Campina Grande PB Macei AL Florianpolis SC Salvador BA Rio Grande RS Belm PA Fortaleza CE Osrio RS Piracicaba SP Recife PE Atibaia SP Goinia GO Boa Vista RR SJ dos Campos SP Nova Iguau RJ Porto Alegre RS Belo Horizonte MG Juazeiro BA Porto Velho RO So Cristovo SE Maring PR S.B. do Campo SP Jata GO

EUREKA! N21, 2005

62

Potrebbero piacerti anche